Science Bowl - Set 3

Pataasin ang iyong marka sa homework at exams ngayon gamit ang Quizwiz!

13) MATH Short Answer Change the following mixed number into an improper fraction: 3 5/18

59/18

19) MATH Short Answer Round the number 590,695 to the nearest hundred:

590,700

14) GENERAL SCIENCE Short Answer Convert the following value into standard scientific notation: 0.00634 × 10^8

6.34 × 10^5

3) MATH Short Answer If 12 gallons of gasoline costs $36.00, what would 1 quart cost, in cents?

75 CENTS

7) PHYSICAL SCIENCE Short Answer How many electrons are in the outer shell of a noble gas with an atomic mass greater than 3?

8

23) MATH Short Answer How many U.S. quarters equal 750 U.S. nickels?

150

9) MATH Short Answer In how many ways can 5 different people be standing around a circular table?

24 (Solution: circular permutation, (n

centimeters high?

48 (Solution: V = l × w × h)

times its height:

5

1) PHYSICAL SCIENCE Short Answer What is the most common scientific term for a change in velocity over time?

ACCELERATION

times as old than Alice. In 6 years from now, Ted will be twice as old as Alice will be. How many years old, respectively, are Alice and Ted today?

ALICE = 6; TED = 18 (Solution: 3x + 6 = 2(x + 6); x = 6 (Alice); 3x = 18 (Ted))

18) PHYSICAL SCIENCE Short Answer How many atoms of each element are in the following chemical compound: Al(NO3)3

Al (Aluminum) = 1; N (Nitrogen) = 3; O (Oxygen) = 9

Simpson scale

BEAUFORT SCALE; SAFFIR

15) GENERAL SCIENCE Short Answer What element is MOST commonly added to steel to make it stainless steel?

CHROMIUM

ahl) zone; abyssal zone

LEAST = ABYSSAL; GREATEST = NERITIC

11) LIFE SCIENCE Short Answer What are the most photosynthetic organs of most higher plants?

LEAVES

10) LIFE SCIENCE Short Answer If the 4 main classes of biological molecules are protein, lipid, carbohydrate, and nucleic acid, which would butter be classified as?

LIPID

wave

P

4) MATH Multiple Choice Which of the following is closest in value to the number 10: W) 9.995 X) 10.01 Y) 9.99 Z) 10.05

W) 9.995

beam balance

W) ALWAYS READ THE UPPER EDGES OF A MENISCUS WHEN DETERMINING VOLUME OF WATER IN A FLASK (Solution: always read the bottom of the meniscus for nearly all solutions)

22) EARTH SCIENCE Multiple Choice Chalk is a type of: W) limestone X) coal Y) rock salt Z) basalt

W) LIMESTONE

14) GENERAL SCIENCE Multiple Choice Which of the following celestial objects is the farthest from Earth: W) the most distant quasar X) the Andromeda galaxy Y) the closest binary star Z) Vega

W) THE MOST DISTANT QUASAR

4) PHYSICAL SCIENCE Multiple Choice Which of the following is NOT true: W) the temperature of a substance changes during a phase change X) a compound can be broken down by chemical means Y) some known elements are unstable and only last for fractions of seconds Z) the opposite of vaporization is condensation

W) THE TEMPERATURE OF A SUBSTANCE CHANGES DURING A PHASE CHANGE

6) PHYSICAL SCIENCE Multiple Choice Which of the following is typically TRUE: W) rare earth metals are also known as alkali metals X) a combustion reaction releases heat Y) noble gases are very reactive elements Z) a reaction in equilibrium cannot move forward or backward

X) A COMBUSTION REACTION RELEASES HEAT

5) PHYSICAL SCIENCE Multiple Choice Which of the following BEST describes the following reaction: 2CaO ! 2Ca + O2 W) combustion X) decomposition Y) single replacement Z) double replacement

X) DECOMPOSITION

waves

X) P

16) LIFE SCIENCE Short Answer What are the scientific terms for the two general types of tissues that conduct fluids in higher plants?

XYLEM AND PHLOEM

8) MATH Multiple Choice Which of the following is NOT true: W) 23 - 3 × 2 = 17 X) 72 ÷ 9 - 3 = 5 Y) 0.3 + 0.4 × 2 = 1.4 Z) 6 + 6 × 2 = 18

Y) 0.3 + 0.4 × 2 = 1.4

grade foliated metamorphic rock: W) geode X) marble Y) gneiss Z) slate

Y) GNEISS

um) Y) radula Z) mantle

Y) RADULA

13) EARTH SCIENCE Multiple Choice Which of the following fossils is the oldest: W) fossil of the earliest flowering plant X) fossilized frog's egg Y) trilobite Z) mastodon

Y) TRILOBITE

4) MATH Multiple Choice Which of the following BEST describes the graph of the following equation, (x - 1)2 + (y + 2)2 = 9 W) straight line X) parabola Y) ellipse Z) circle

Z) CIRCLE

sond) balloons

Z) RADIOSONDE BALLOONS

9) MATH Short Answer A cell phone company charges 25 cents per minute for the first 10 minutes of usage and 10 cents per minute for usage greater than 10 minutes. If you talked for 1½ hours, what is your total airtime charge?

$10.50 (Solution: 0.25 × 10 = $2.50; 80 min × 0.1 = $8.00; $8 + $2.5 = $10.50)

14) MATH Short Answer What is the total price of admission to the theater for 2 adults and 4 children if the ticket price for adults is $30.00 and for children is $18.00?

$132.00

2, 1) and the midpoint is at (8, 1), find the other endpoint:

(18, 1)

22) MATH Short Answer Factor the following polynomial over the integers: 49x 2 - 64

(7x + 8)(7x - 8)

1) MATH Short Answer Three points on a graph are connected by lines to form a triangle. If the center of dilation is the origin and one of the points is (4, 6), what is the new point for a dilation factor of 2?

(8, 12)

15) MATH Short Answer Find the midpoint of the line segment joining the following pair of points: (4, 1) and (12, 7)

(8, 4) (Solution: midpoint x = (x1 + x2)/2 = (4 + 12)/2 = 8; midpoint y = (y1 + y2)/2 = ( 1 + 7)/2 = 4) 15) MATH Short Answer If one endpoint of a given line segment is at point (

3) MATH Short Answer Provide the prime factors over the integers for the following expression: x 3 + 8

(x + 2) and (x 2 - 2x + 4) (DO NOT ACCEPT (x + 2) times (x 2 - 2x + 4))

24) MATH Short Answer In simplest terms, give the 6th term of the geometric sequence whose second term is -3 and whose third term is 2 3 :

- 16 3 (Solution: ax = a(r)x

MATH Short Answer Arrange the following in increasing order: (3)^2, -(4)^2, (1)^4, -(-3)^3

-(4)^2, (-1)^4, (-3)^2, -(-3)^3

25) MATH Short Answer Evaluate the following radical expression: - 16 (read as: the negative square root of the square root of 16)

-2

13) MATH Short Answer Evaluate the following expression: -6 × 3 + -16 ÷ 4

-22

23) MATH Short Answer If the discriminant, of a quadratic equation is negative, how many real solutions does the equation have?

0

14) MATH Short Answer Giving your answer as a decimal number, what does 10 to the minus three power equal?

0.001

5) GENERAL SCIENCE Short Answer Convert 300 milliliters into liters:

0.3

16) PHYSICAL SCIENCE Short Answer What is the ideal mechanical advantage of a single fixed pulley?

1

6) MATH Short Answer Giving your answer as a mixed fraction reduced to its lowest terms, subtract the following fractions: (8/9-1/9)-(5/9-12/9)

1 5/9

1) PHYSICAL SCIENCE Short Answer By number or words, order the following 3 choices from the one that travels the SLOWEST to the FASTEST: 1) sound in 0ºC air; 2) electromagnetic radiation; 3) a spaceship at 5,000 meters per second

1, 3 , 2 (ACCEPT: SOUND IN 0ºC AIR; A SPACESHIP AT 5,000 METERS PER SECOND; ELECTROMAGNETIC RADIATION)

5) MATH Short Answer If 2 fair dice are rolled simultaneously, what is the probability that the 2 dice will show the same number expressed as a fraction in lowest terms?

1/6

18) MATH Short Answer A rectangular water storage tank measures 30 meters long, 10 meters wide and 12 meters deep. If water is leaking out at a constant rate of 0.5 cubic meters per minute, how many hours will it take for the water level to drop one meter?

10 (Solution: 300m3 /(0.5m3 /min) = 600 min/(60 min/hr) = 10 hrs)

23) MATH Short Answer Find the 21st term in the arithmetic sequence whose second term is -5 and fourth term is 7:

109

3) MATH Short Answer How many faces does a dodecagon have?

12

3) MATH Short Answer Giving your answer in meters to the first decimal place, if one leg of an isosceles right triangle is 10 meters long, how long is the hypotenuse?

14.1

2) GENERAL SCIENCE Short Answer To the nearest whole number, convert 60ºC into degrees Fahrenheit:

140

22) MATH Short Answer Consider a distribution of 7 scores with a median of 20. If the lowest score value is decreased by 4, what will the new median value be for this distribution of 7 scores?

20

10) LIFE SCIENCE Short Answer How many chromosomes does a mature human sperm typically have?

23

sex cell and 2s is the DNA content of the same cell, what would be the values of n and s at the beginning of metaphase in mitosis?

2n AND 4s

20) MATH Short Answer How many different outcomes are possible if eight hydrogen fuel cars compete in a race for which first, second, and third prizes will be awarded and no ties are possible?

336 (Solution: P(8,3) = 8!/(8

19) MATH Short Answer If two sides of a parallelogram are 8 and 9 centimeters, respectively, find the perimeter, in centimeters:

34

24) MATH Short Answer If the fourth term in the binomial expansion of (x + y) 7 is 35x 4 y 3 , what is the 5th term?

35x 3 y 4 (ACCEPT: 35y 4 x 3 )

21) MATH Short Answer What is the area of a square, in square meters, if its perimeter is 28 meters?

49

22) PHYSICAL SCIENCE Short Answer How many grams of potassium chloride, with a molecular weight of 104, must be dissolved in 250 milliliters of water to make a 20% potassium chloride solution?

50 (Solution: (250ml) × (20g/100ml) = 50g)

9) MATH Short Answer Giving your answer to the second decimal place, add the following three numbers: 47 + 4.7 + 0.47

52.17

1) MATH Short Answer Giving your answer in terms of π, what is the length of a 36º arc in a circle that has a circumference of 50π?

5π (Solution: C = 50π; 1/10th of 50π = 5π)

10) PHYSICAL SCIENCE Short Answer How many grams of sodium chloride are in 300 grams of a 20% solution, if the atomic masses of sodium and chloride are 23 and 35, respectively?

60

20) MATH Short Answer If one of 2 complementary angles is 32º greater than the other, how many degrees is the larger angle?

61

24) GENERAL SCIENCE Short Answer Your boat shows a compass direction of 61 degrees and your navigation chart shows a variation of 12 degrees east. If you know your deviation is zero, what is your true course, in degrees?

73

19) MATH Short Answer Find the number of sides in a regular polygon if each interior angle measures 135 degrees:

8 (Solution: (n - 2)180º = 135n; n = 8)

22) GENERAL SCIENCE Short Answer When all other things are held constant and a star is1/3 as far from an observer as it originally was, how many times as bright would it appear?

9

5) MATH Short Answer Give the numerical value of 5 factorial minus 4 factorial:

96

12) EARTH SCIENCE Short Answer An alpine glacier generally has 2 areas, an accumulation zone and an area of net loss where the previous winter's snow melts away. What is the specific name for this zone?

ABLATION (ACCEPT: ABLATION ZONE or ABLATION AREA)

25) LIFE SCIENCE Short Answer Which 2 of the following 5 molecules are most directly involved in muscle contraction in mammals: cellulose; albumin; actin; collagen; myosin

ACTIN; MYOSIN

25) LIFE SCIENCE Short Answer What specific human gland is the primary production site of epinephrine?

ADRENAL GLAND

13) GENERAL SCIENCE Short Answer From what continent do more than 80% of the weather systems originate which become tropical storms or hurricanes that reach the U.S. eastern and southern shores?

AFRICA

17) PHYSICAL SCIENCE Short Answer Name all of the following 3 forms of radioactive decay that change the atomic number of the decaying element: alpha decay; beta decay; gamma decay

ALPHA DECAY; BETA DECAY

16) GENERAL SCIENCE Short Answer Name all of the following 5 substances that are NOT derived from petroleum: amber; kerosene; asphalt; latex; paraffin

AMBER; LATEX

10) EARTH SCIENCE Short Answer What is the most common scientific name of the instrument used to measure wind speed?

ANEMOMETER

9) GENERAL SCIENCE Short Answer Name all of the following 4 organisms that are typically filter feeders in an aquatic environment: barnacles; corals; frogs; kelp

BARNACLES, CORALS

23) GENERAL SCIENCE Short Answer What is the most common name of the object in outer space that has a gravitational field so strong that light cannot escape from it?

BLACK HOLE

21) PHYSICAL SCIENCE Short Answer Distillation depends mostly upon what physical property?

BOILING POINT (ACCEPT: CONDENSATION POINT)

21) PHYSICAL SCIENCE Short Answer What gas law states that the volume of a gas is inversely proportional to its pressure as long as the temperature and number of particles are constant?

BOYLE'S (LAW)

10) GENERAL SCIENCE Short Answer The amount of energy stored in food is commonly measured in what unit?

CALORIE

5) LIFE SCIENCE Short Answer Found in humans and quite prominent in dogs, what is the name of the conical-shaped tooth used in tearing muscle and other foods and assisting in cutting?

CANINE

1) LIFE SCIENCE Short Answer Of the 4 general types of biological molecules, proteins, carbohydrates, lipids and nucleic acids, which one typically accounts for the majority of dietary calories in the human diet?

CARBOHYDRATES

15) PHYSICAL SCIENCE Short Answer Name all of the following 3 pairs of elements that would be chemically bonded by a covalent bond: carbon and carbon; magnesium and chlorine; silicon and oxygen

CARBON AND CARBON; SILICON AND OXYGEN

3) PHYSICAL SCIENCE Short Answer Butane is made up of what 2 elements?

CARBON AND HYDROGEN (ACCEPT: C AND H)

22) GENERAL SCIENCE Short Answer What gaseous molecule is produced from the burning of fossil fuels and is believed to be the primary cause of the greenhouse warming effect on Earth?

CARBON DIOXIDE (ACCEPT: CO2)

17) LIFE SCIENCE Short Answer Plant cell walls are made primarily of microfibers of what common polysaccharide?

CELLULOSE

20) GENERAL SCIENCE Short Answer What natural polymer is the main component of cotton?

CELLULOSE

6) LIFE SCIENCE Short Answer What is the scientific name for the instrument used to separate substances of different densities in solution using a rotor to spin specimens at high RPM's?

CENTRIFUGE

6) LIFE SCIENCE Short Answer Arachnids have 4 pairs of walking legs and a single pair of pedipalps. What is the name of the last pair of appendages?

CHELICERA

11) LIFE SCIENCE Short Answer What higher plant cell organelle is responsible for using sunlight to convert carbon dioxide and water into sugar and oxygen?

CHLOROPLAST

25) LIFE SCIENCE Short Answer What is the most common name for the polymers of DNA and associated proteins that are found in the eukaryotic nucleus?

CHROMOSOME

18) EARTH SCIENCE Short Answer Which one of the following 5 choices is NOT a mineral: diamond; asbestos; graphite; coal; talc

COAL

25) LIFE SCIENCE Short Answer Which one of the following 5 molecules is a fibrous molecule that is very common in locations in between cells of the human body: cellulose; albumin; insulin; collagen; hemoglobin

COLLAGEN

ah)?

CONES

5) GENERAL SCIENCE Short Answer What is the name for the manipulated variable group in all experiments that is compared to the experimental group?

CONTROL

20th century by exposure to what outlawed insecticide?

DDT (ACCEPT: DICHLORO

20) GENERAL SCIENCE Short Answer What is the name for the muscle that contracts when you inhale and causes your chest cavity to expand?

DIAPHRAGM

7) PHYSICAL SCIENCE Short Answer The overall charge of an atom of a certain element depends on the gaining or losing of what sub-atomic particles?

ELECTRONS

8) EARTH SCIENCE Short Answer What is the name for the location on the Earth's surface directly above an earthquake focus?

EPICENTER

4) LIFE SCIENCE Short Answer Order the following from the part nearest to the START of the human digestive system to the part nearest the END: ileum; esophagus; duodenum; jejunum

ESOPHAGUS; DUODENUM; JEJUNUM; ILEUM

3) EARTH SCIENCE Short Answer What is the name for the internal ring within a hurricane where the most intense squalls occur?

EYEWALL

2) LIFE SCIENCE Short Answer Which one of the following 5 organisms is LEAST related to the others: mites; spiders; scorpions; fleas; ticks

FLEAS

8) PHYSICAL SCIENCE Short Answer What is the most common term used to describe why astronauts orbiting the Earth experience apparent weightlessness?

FREE FALL

19) LIFE SCIENCE Short Answer What insect is used the most in experiments studying genetics and development?

FRUIT FLY (ACCEPT: DROSOPHILA MALANOGASTER OR DROSOPHILA)

UP 20) MATH Short Answer What is the greatest common factor and the least common multiple of the following 3 numbers, respectively: 3, 6, and 8

GCF = 1; LCM = 24 BONUS 20) MATH Short Answer Find the product of the following 2 binomials, giving your answer in standard form: (-x - 3y)(-x + 3y) ANSWER: x^2

1) LIFE SCIENCE Short Answer Starch and glycogen are polymers of what simple sugar?

GLUCOSE

10) PHYSICAL SCIENCE Short Answer Cellulose is a polymer of what 6-carbon sugar?

GLUCOSE

13) LIFE SCIENCE Short Answer What is the most common simple sugar or monosaccharide that makes up most of the carbohydrates of the typical American's diet?

GLUCOSE

17) LIFE SCIENCE Short Answer What cellular organelle functions to modify and package proteins it received from the rough endoplasmic reticulum and will typically prepare them for exporting outside the cell?

GOLGI (ACCEPT GOLGI BODY OR GOLGI APPARATUS)

12) EARTH SCIENCE Short Answer Of the following 5 choices, name 2 that are igneous rocks: rock salt; granite; basalt; sandstone; gypsum

GRANITE AND BASALT

22) GENERAL SCIENCE Short Answer A forest fire in part of the New York Adirondack mountains wipes out all existing vegetation. Arrange the following 4 choices from the FIRST that is most likely to repopulate the land to the LAST to repopulate: pine trees; scrubs; oak trees; grasses

GRASSES; SCRUBS; PINE TREES; OAK TREES

15) LIFE SCIENCE Short Answer Geotropism is a response of plants to what external force?

GRAVITY

22) EARTH SCIENCE Short Answer Which 2 of the following 5 choices are sedimentary rocks formed as the result of evaporation and precipitation: gypsum; marble; rock salt; gneiss; diorite

GYPSUM; ROCK SALT BONUS 22) EARTH SCIENCE Short Answer As viewed from Earth, what is the proper name for the brightest

22) LIFE SCIENCE Short Answer Order the following 4 human ear structures from the OUTERMOST to the INNERMOST: anvil; stapes; hammer; oval window

HAMMER; ANVIL; STAPES; OVAL WINDOW

24) PHYSICAL SCIENCE Short Answer What is the most common scientific term for the back and forth oscillating motion of a swinging pendulum?

HARMONIC (ACCEPT: SIMPLE HARMONIC MOTION)

5) LIFE SCIENCE Short Answer The grasshopper body is grossly divided into what 3 parts?

HEAD; THORAX; ABDOMEN (in any order)

24) PHYSICAL SCIENCE Short Answer What is the SI the unit for frequency?

HERTZ

19) MATH Short Answer Order the following 4 polygons from the one with the FEWEST faces to the one with the MOST faces: dodecagon; hexadecagon; octagon; hexagon

HEXAGON; OCTAGON; DODECAGON; HEXADECAGON

23) GENERAL SCIENCE Short Answer Both the Sun and the Gas Giants in our Solar System are composed primarily of what 2 elements?

HYDROGEN AND HELIUM

22) PHYSICAL SCIENCE Short Answer What three elements are found in ethanoic acid?

HYDROGEN; CARBON; OXYGEN (in any order)

17) GENERAL SCIENCE Short Answer Arrange the following 4 organisms from the first to appear on Earth to the most recent to appear on Earth: fish; invertebrates; reptiles; mammals

INVERTEBRATES; FISH; REPTILES; MAMMALS

11) PHYSICAL SCIENCE Short Answer Which 2 of the following 4 elements are most easily magnetized at room temperature: iron; copper; nickel; aluminum

IRON AND NICKEL

8) EARTH SCIENCE Short Answer Which 2 of the following 5 elements is the Earth's outer core thought to be primarily made of: aluminum; sodium; iron; silicon; nickel

IRON AND NICKEL (in any order)

25) LIFE SCIENCE Short Answer What is the primary protein from which eagles talons are made?

KERATIN

2) EARTH SCIENCE Short Answer What is the specific name for ridges of the giant accumulations of glacial till that form along the outer sides or edges of an alpine glacier?

LATERAL MORAINES (DO NOT ACCEPT: MORAINE)

11) MATH Short Answer Of the following five quantities, which is the LEAST and which is the GREATEST, respectively: : 0.425; ¼; 2(squareroot) ; - 9(squareroot) ; 4/3

LEAST = - 9 (squareroot); GREATEST = 2 (squareroot)

24) LIFE SCIENCE Short Answer Bile is produced by what organ of the human body?

LIVER

15) MATH Short Answer What was the net gain or loss in yardage for a football team that in four straight plays gained 12 yards, then lost 9 yards, gained 3 yards, and finally lost 23 yards?

LOSS OF 17

20) EARTH SCIENCE Short Answer What is the name for the layer of the Earth that is nearly 1,800 miles thick and represents about 68% of the Earth's mass?

MANTLE

25) GENERAL SCIENCE Short Answer The lifetime of a star is mostly determined by what physical property?

MASS

3) EARTH SCIENCE Short Answer Name all the terrestrial planets from the one that is CLOSEST to the Sun to the one that is FARTHEST from the Sun:

MERCURY, VENUS, EARTH, MARS

18) LIFE SCIENCE Short Answer What type of nucleic acid has codons for matching with anticodons?

MESSENGER RNA (ACCEPT: mRNA)

19) PHYSICAL SCIENCE Short Answer What are the most common SI units used for the acceleration of an object?

METERS PER SECOND SQUARED

15) LIFE SCIENCE Short Answer Name all of the following 5 plants or plant groups that produce true flowers: sphagnum moss; fern; monocots; pine tree; cactus

MONOCOTS AND CACTUS

13) PHYSICAL SCIENCE Short Answer What atomic particle is most directly responsible for fissioning each nucleus in an atomic chain reaction?

NEUTRON

sea divers if they swim to the surface too quickly?

NITROGEN

9) LIFE SCIENCE Short Answer The basic parts of ribosomes are manufactured in what specific part of a eukaryotic cell?

NUCLEOLUS (DO NOT ACCEPT: NUCLEUS)

6) PHYSICAL SCIENCE Short Answer Name all of the following 4 choices that contain ionic bonds: NaCl; O2; KBr; CO2

NaCl; KBr

4) PHYSICAL SCIENCE Short Answer Name all of the following 4 substances that are bases: MgCl; NaOH; HCl; H2SO4

NaOH (ACCEPT: SODIUM HYDROXIDE)

8) MATH Short Answer In what quadrant will a 1135º angle lie, assuming it is in standard position?

ONE (ACCEPT: FIRST)

8) LIFE SCIENCE Short Answer What is the scientific term for the type of circulatory system found in lobsters in which blood empties into spaces in the body that are not contained in vessels?

OPEN

10) LIFE SCIENCE Short Answer What is the anatomical term for the nerves in humans that conduct messages from the eyes to the brain?

OPTIC

23) LIFE SCIENCE Short Answer What is the most common term for the arrangement of veins in the leaves of most monocots?

PARALLEL

5) PHYSICAL SCIENCE Short Answer Order the following 3 particles from the one with the LEAST mass to the one with the MOST mass: electron; neutron; photon

PHOTON; ELECTRON; NEUTRON

3) LIFE SCIENCE Short Answer Order the following 4 groups from the LARGEST taxonomical group to the SMALLEST taxonomical group: family; class; order; phylum

PHYLUM; CLASS; ORDER; FAMILY

21) GENERAL SCIENCE Short Answer Order the following 4 metric units from the SMALLEST to the LARGEST: microgram; nanogram; picogram; milligram

PICOGRAM; NANOGRAM; MICROGRAM; MILLIGRAM

17) PHYSICAL SCIENCE Short Answer Of the 4 states of matter, which one is the most common in the universe?

PLASMA

white when pure, is used as a catalyst in fuel cells and in antipollution devices in automobiles, does not oxidize in air at any temperature, and is used extensively in the making of jewelry and wires?

PLATINUM

3) PHYSICAL SCIENCE Short Answer Order the following 3 elements from the one that most easily loses an electron to the one which will be the most difficult to remove an electron from: potassium; fluorine; nitrogen

POTASSIUM; NITROGEN; FLUORINE

17) EARTH SCIENCE Short Answer If you could dive from the ocean surface to the abyssal plain, name all of the following 4 choices that would typically be INCREASED at the abyssal plain: pressure; density; salinity; temperature

PRESSURE; DENSITY; SALINITY (ACCEPT: ALL BUT TEMPERATURE)

11) LIFE SCIENCE Short Answer Order the following 4 stages of mitosis from the EARLIEST to that LATEST: metaphase; prophase; telophase; anaphase

PROPHASE; METAPHASE; ANAPHASE; TELOPHASE

20) LIFE SCIENCE Short Answer Of the 4 general types of biological molecules: proteins, carbohydrates, lipids and nucleic acids, under which one would most enzymes be categorized?

PROTEINS

14) LIFE SCIENCE Short Answer What structure of the human eye is primarily responsible for regulating the amount of light that enters the eye?

PUPIL

14) PHYSICAL SCIENCE Short Answer Name all of the following 3 forms of energy transfer that move in the form of electromagnetic waves: conduction; convection; radiation

RADIATION

17) PHYSICAL SCIENCE Short Answer In the reaction sodium plus fluorine yields sodium fluoride, what are the reactants and products, respectively?

REACTANTS = SODIUM AND FLUORINE; PRODUCTS = SODIUM FLUORIDE

9) LIFE SCIENCE Short Answer What cellular organelle most directly functions to help match the tRNA to the correct place on the mRNA and allows the growing of a protein?

RIBOSOME

half of all humans on Earth: corn; barley; wheat; rice; sorghum

RICE

2) LIFE SCIENCE Short Answer When blood returns to the human heart from the lower part of the body, to what specific chamber will the blood first enter?

RIGHT ATRIUM (DO NOT ACCEPT: ATRIUM)

northern latitudes, if the Moon's phase is a waxing crescent?

RISE = DAWN; SET = SUNSET

90 degree right triangle if the length of the hypotenuse is 12 centimeters?

SHORTER LEG = 6; LONGER LEG = 6 3

25) GENERAL SCIENCE Short Answer Today, most semiconductors and transistors are made mostly with what element?

SILICON

2) PHYSICAL SCIENCE Short Answer Silicon dioxide is made of what two elements?

SILICON AND OXYGEN

appearing star in the night sky of mid northern latitudes?

SIRIUS

24) EARTH SCIENCE Short Answer Which one of the following 5 choices is a metamorphic rock: tuff; obsidian; slate; gabbro; peridotite

SLATE

24) LIFE SCIENCE Short Answer Into what human organ does the gall bladder release bile?

SMALL INTESTINE (ACCEPT: DUODENUM)

23) PHYSICAL SCIENCE Short Answer Arrange the four states of matter in order of INCREASING kinetic energies for a given material:

SOLID; LIQUID; GAS; PLASMA

24) PHYSICAL SCIENCE Short Answer Solutions with a pH less than about 6 typically have what distinctive taste?

SOUR

3) EARTH SCIENCE Short Answer What energy source is the primary driver of the Earth's water cycle?

SUN

21) EARTH SCIENCE Short Answer Order the following 4 minerals from the SOFTEST to the HARDEST: fluorite; gypsum; talc; calcite

TALC; GYPSUM; CALCITE; FLUORITE

12) LIFE SCIENCE Short Answer What specific type of nucleic acid carries an amino acid to a growing protein?

TRANSFER RNA (ACCEPT: tRNA)

24) EARTH SCIENCE Short Answer Arrange the following 4 layers of the Earth's atmosphere in order of DECREASING air pressure: stratosphere; exosphere; thermosphere; troposphere

TROPOSPHERE; STRATOSPHERE; THERMOSPHERE; EXOSPHERE

19) GENERAL SCIENCE Short Answer Of the following 4 choices, name all that are amino acids: tryptophan; testosterone; cholesterol; niacin

TRYPTOPHAN

20) LIFE SCIENCE Short Answer What general type of human blood vessel returns blood to the heart?

VEIN

10) GENERAL SCIENCE Short Answer What planet in our Solar System is about the same size as the Earth, is tectonically active, and has a thick atmosphere composed of carbon dioxide?

VENUS

22) EARTH SCIENCE Short Answer On which of the terrestrial planets does the Sun rise in the west?

VENUS

5) EARTH SCIENCE Short Answer At its maximum brightness, which planet of our solar system appears brighter than any other planet in the sky?

VENUS

7) PHYSICAL SCIENCE Short Answer Which one of the following 4 colors of light has the greatest angle of refraction in normal glass: orange; violet; red; yellow

VIOLET

to the third power

W) -(-k) 3

5) GENERAL SCIENCE Multiple Choice Which of the following is closest to average barometric pressure at sea level, in millibars: W) 1013 X) 760 Y) 10 Z) 1

W) 1013

2) PHYSICAL SCIENCE Multiple Choice Which of the following has the greatest density at room temperature: W) 2 kilograms of uranium X) 3 kilograms of calcium Y) 4 kilogram of lithium Z) 5 kilograms of ice

W) 2 KILOGRAMS OF URANIUM

15) LIFE SCIENCE Multiple Choice Which of the following is a common excitatory neurotransmitter in mammals: W) acetylcholine X) insulin Y) estrogen Z) glycine

W) ACETYLCHOLINE

19) PHYSICAL SCIENCE Multiple Choice Because they contain mercury, which of the following is the BEST classification of silver dental fillings: W) amalgam X) colloid Y) mixture Z) solution

W) AMALGAM (Solution: an amalgam is an alloy of mercury)

2) PHYSICAL SCIENCE Multiple Choice Which of the following is the BEST example of a chemical reaction: W) an acid is added to a clear solution in a test tube and a white substance appears and falls to the bottom of the test tube X) milk is added to water and the water turns a milky color Y) molten metal hardens upon cooling Z) salt is dissolved in water

W) AN ACID IS ADDED TO A CLEAR SOLUTION IN A TEST TUBE AND A WHITE SUBSTANCE APPEARS AND FALLS TO THE BOTTOM OF THE TEST TUBE

1) MATH Multiple Choice Which of the following BEST describes the number property used in the following equation: (3 × 2) × 5 = 3 × (2 × 5) W) associative property of multiplication X) commutative property of multiplication Y) distributive property of multiplication Z) multiplicative property of one

W) ASSOCIATIVE PROPERTY OF MULTIPLICATION

pressure weather system is typically associated with which of the following conditions: W) cool with high thin clouds X) cool with nimbostratus clouds Y) warm with moderate overcast Z) warm with cumulonimbus clouds

W) COOL WITH HIGH THIN CLOUDS

7) PHYSICAL SCIENCE Multiple Choice Which of the following functional groups replaces a hydrogen in an alcohol: W) hydroxyl X) carboxyl Y) ester Z) amide

W) HYDROXYL

Atlantic ridge Z) direct sampling and analysis of mantle material

W) PALEOMAGNETIC DATA

tropical Z) tropical

W) POLAR

ruler and you get the following 3 results: 2.99 inches, 3.00 inches, and 3.01 inches. If the object's actual length is 3.28 inches, then the data is: W) precise but not accurate X) accurate but not precise Y) both accurate and precise Z) neither accurate nor precise

W) PRECISE BUT NOT ACCURATE

3) LIFE SCIENCE Multiple Choice Which of the following is NOT a rodent: W) rabbit X) chipmunk Y) prairie dog Z) squirrel

W) RABBIT

15) EARTH SCIENCE Multiple Choice Which of the following is the BEST example of a littoral ocean current: W) rip current X) seismic sea wave Y) the Gulf Stream Z) equatorial tidal current

W) RIP CURRENT

14) EARTH SCIENCE Multiple Choice Which of the following is NOT true: W) the Milky Way is an elliptical galaxy X) our Sun will become a red giant in the far distant future Y) constellations are patterns of stars in the night sky Z) the average sunspot cycle is 11 years

W) THE MILKY WAY IS AN ELLIPTICAL GALAXY

14) PHYSICAL SCIENCE Multiple Choice Which of the following MUST be the case for a more massive object versus a less massive object if both are at the same temperature and pressure: W) the more massive object will have more thermal energy X) the more massive object will have a greater specific heat Y) the more massive object will lose heat faster than the smaller object Z) the more massive object will have less inertia

W) THE MORE MASSIVE OBJECT WILL HAVE MORE THERMAL ENERGY

24) MATH Multiple Choice Which of the following is a quadrilateral with only one pair of parallel lines: W) trapezoid X) rhombus Y) rectangle Z) hexagon

W) TRAPEZOID

9) GENERAL SCIENCE Short Answer What molecular substance is typically considered the most critical for life to exist on another planet?

WATER

2) LIFE SCIENCE Short Answer Oxygen released by plants during photosynthesis originates from what substance?

WATER (ACCEPT: H2O)

11) PHYSICAL SCIENCE Short Answer Order the following 3 liquids from the one with the LOWEST viscosity to the one with the HIGHEST viscosity at room temperature: pure olive oil; water; pure honey

WATER, PURE OLIVE OIL, PURE HONEY

15) GENERAL SCIENCE Multiple Choice Which of the following metric masses are NOT equal: W) 1000 milligrams = 1 gram X) 100 milligrams = 1 centigram Y) 100 decigrams = 1 decagram Z) 100 grams = 1 hectogram

X) 100 MILLIGRAMS = 1 CENTIGRAM

ounce soda can: W) 6 centimeters X) 12 centimeters Y) 18 centimeters Z) 24 centimeters

X) 12 CENTIMETERS

thaw cycles X) a glacier once moved in the direction of these scratches Y) a swift river once ran through these areas Z) the area was once a pluvial lake

X) A GLACIER ONCE MOVED IN THE DIRECTION OF THESE SCRATCHES

19) GENERAL SCIENCE Multiple Choice Thyroid disease is typically thought of as: W) a blood disorder X) a metabolic disease Y) a digestive system disorder Z) an immune system disorder

X) A METABOLIC DISEASE

25) GENERAL SCIENCE Multiple Choice Which of the following elements acts as a catalyst for destroying ozone in the upper atmosphere: W) sulfur X) chlorine Y) nitrogen Z) helium

X) CHLORINE

smelling flowers: W) amines X) esters Y) alcohols Z) halogenated hydrocarbons

X) ESTERS

dominant for white and red

X) HETEROZYGOUS

16) PHYSICAL SCIENCE Multiple Choice Real images produced by a single lens are always: W) upright X) inverted Y) virtual Z) smaller

X) INVERTED

17) LIFE SCIENCE Multiple Choice Which of the following types of nucleic acid carries a complementary copy of a gene from the DNA to the cytoplasm to instruct the order in which amino acids will be made into a protein: W) transfer RNA X) messenger RNA Y) ribosomal RNA Z) DNA

X) MESSENGER RNA

12) GENERAL SCIENCE Multiple Choice Which of the following typically accounts for the greatest amount of magnification in a compound light microscope: W) condenser lens X) objective lens Y) ocular lens Z) diaphragm

X) OBJECTIVE LENS

9) PHYSICAL SCIENCE Multiple Choice Which of the following metals is often used to transform a sheet of regular glass into a mirror: W) cobalt X) silver Y) zinc Z) iron

X) SILVER

thickness burn: W) 1st degree X) 2nd degree Y) 3rd degree Z) 4th degree

Y) 3rd DEGREE

4) EARTH SCIENCE Multiple Choice Which of the following terms is LEAST related to the action of glaciers: W) esker X) terminal moraine Y) karst topography Z) kettle lake

Y) KARST TOPOGRAPHY

range downwind as far away as New England Z) most of the U.S. and Canada

Y) LONG

waves

Y) ONLY P

6) EARTH SCIENCE Multiple Choice Which of the following is TRUE: W) pyroclastic flows dominate Hawaiian volcanoes X) granitic magma contains less silica than basaltic magma Y) peridotite is rock found within the Earth's mantle and some meteorites Z) plutons are large masses of basalt

Y) PERIDOTITE IS ROCK FOUND WITHIN THE EARTH'S MANTLE AND SOME METEORITES

11) GENERAL SCIENCE Multiple Choice Which of the following is a polymer commonly used to manufacture plastic bottles, toys and indoor/outdoor carpeting: W) polyvinyl chloride X) polystyrene Y) polypropylene Z) nylon

Y) POLYPROPYLENE

8) EARTH SCIENCE Multiple Choice Which of the following is the global wind system that most influences weather in Denver Colorado: W) tropical easterlies X) trade winds Y) prevailing westerlies Z) polar easterlies

Y) PREVAILING WESTERLIES

12) PHYSICAL SCIENCE Multiple Choice Which of the following can transfer heat through a vacuum: W) convection X) conduction Y) radiation Z) sublimation

Y) RADIATION

times greater than the Moon's influence Z) each lunar day, some coastlines have 2 high tides and 2 low tides whereas some have only have 1 high tide and 1 low tide

Y) THE SUN'S INFLUENCE ON EARTH'S TIDES IS ABOUT 2

18) LIFE SCIENCE Multiple Choice Which of the following is the LEAST likely method a maple tree uses to regulate water: W) storing water in vacuoles X) opening and closing stomata in stems and leaves Y) transporting water in their heartwood Z) absorbing water through root hairs

Y) TRANSPORTING WATER IN THEIR HEARTWOOD

21) GENERAL SCIENCE Multiple Choice In which of the following biomes are you MOST likely to find permafrost: W) taiga X) deciduous forest Y) tundra Z) chaparral

Y) TUNDRA

preserved dinosaur fossil: W) within key beds of the Devonian Period X) within metamorphic rocks of the Cretaceous Period Y) within sedimentary rocks laid down during the Mesozoic Era Z) within volcanic casts of the Jurassic Period

Y) WITHIN SEDIMENTARY ROCKS LAID DOWN DURING THE MESOZOIC ERA

7) EARTH SCIENCE Multiple Choice In which of the following locations would you typically expect the average salinity of surface sea water to be the LOWEST: W) in the Dead Sea X) in the waters off Antarctica Y) in the Chesapeake Bay Z) at the mouth of the Mississippi River

Z) AT THE MOUTH OF THE MISSISSIPPI RIVER

20) LIFE SCIENCE Multiple Choice Which of the following trees have simple, pinnate leaves: W) red oak and fig X) sugar maple and sycamore Y) sassafras and palm Z) cherry and apple

Z) CHERRY AND APPLE

24) GENERAL SCIENCE Multiple Choice Pheromones are chemicals produced by various species and function primarily in: W) camouflage X) digestion Y) sweat secretion Z) communication

Z) COMMUNICATION

2) EARTH SCIENCE Multiple Choice Which of the following is the process by which water vapor changes directly into rain: W) sublimation X) evaporation Y) transpiration Z) condensation

Z) CONDENSATION

shaped valley Z) drumlin

Z) DRUMLIN

4) GENERAL SCIENCE Multiple Choice Which of the following is a coniferous tree: W) poplar X) oak Y) elm Z) fir

Z) FIR

17) PHYSICAL SCIENCE Multiple Choice Which of the following is the scientific term for the energy needed to transform a given quantity of a substance from a liquid to a gas at standard pressure: W) heat of boiling X) heat of fusion Y) heat of transition Z) heat of vaporization

Z) HEAT OF VAPORIZATION

12) EARTH SCIENCE Multiple Choice Which of the following is the Epoch name given to the last approximately 10,000 years of Earth's history: W) Quaternary X) Eocene Y) Miocene Z) Holocene

Z) HOLOCENE

axis, its slope is 1

Z) IF A LINE IS PARALLEL TO THE X

11) LIFE SCIENCE Multiple Choice As a tree ages, which of the following BEST describes what happens to the secondary phloem: W) it stops being produced X) it accumulates in large quantities into the xylem Y) it is crushed between each year's xylem Z) it is lost as part of the bark and does not accumulate in great quantities

Z) IT IS LOST AS PART OF THE BARK AND DOES NOT ACCUMULATE IN GREAT QUANTITIES

2) LIFE SCIENCE Multiple Choice Which of the following is the BEST explanation for what would happen to a red blood cell if it were placed in a 25% sugar solution: W) it would lose salt to the sugar solution and gain sugar and remain the same size X) it would gain salt and swell Y) it would lose water to the sugar solution and gain salt and remain the same size Z) it would lose water to the sugar solution and shrink

Z) IT WOULD LOSE WATER TO THE SUGAR SOLUTION AND SHRINK

24) EARTH SCIENCE Multiple Choice Which of the following is NOT true about rocks: W) layering is characteristic of sedimentary rock X) during metamorphosis, limestone becomes marble Y) chalk is primarily made up of microscopic shells Z) quartz reacts with dilute hydrochloric acid to give off carbon dioxide

Z) QUARTZ REACTS WITH DILUTE HYDROCHLORIC ACID TO GIVE OFF CARBON DIOXIDE

19) GENERAL SCIENCE Multiple Choice Which of the following household objects is an example of a heat pump: W) electric baseboard heater X) plasma TV Y) microwave Z) refrigerator

Z) REFRIGERATOR

11) PHYSICAL SCIENCE Multiple Choice Which of the following is NOT true concerning pure liquid water: W) it is a good solvent for simple sugars X) it has a high specific heat Y) it is a requirement for all known life forms Z) the colder it gets the denser it gets from 100ºC to 0ºC

Z) THE COLDER IT GETS THE DENSER IT GETS FROM 100ºC TO 0ºC

oceanic ridge

Z) THE VAST MAJORITY OF VOLCANOES ARE FOUND ALONG THE MIDOCEANIC RIDGE

8) LIFE SCIENCE Multiple Choice Which of the following is the BEST explanation for the function of nuclear pores: W) to prevent damaging hydrogen peroxide from entering the nucleus X) to prevent too much pressure from metabolism from building up in the nucleus Y) to allow the nucleus to sweat Z) to allow substances such as mRNA to leave the nucleus

Z) TO ALLOW SUBSTANCES SUCH AS mRNA TO LEAVE THE NUCLEUS

soluble vitamin

Z) VITAMIN E IS A WATER

9) EARTH SCIENCE Multiple Choice Which of the following is the most immediate driving force of surface ocean currents such as the North Pacific Drift and Gulf Stream currents: W) tides X) centripetal force of the oceans Y) daily temperature variations Z) winds

Z) WINDS

LIFE SCIENCE Multiple Choice An intracellular substance is so named because it is located in which of the following places: W) inside and outside of cells X) underneath cells Y) outside of cells Z) within cells

Z) WITHIN CELLS

13) MATH Short Answer Express the following radical in its simplest radical form if all variables represent positive quantities: 5 (16 ) 2 4 a b c

ab c 2 20

25) MATH Short Answer Divide the polynomial, x 2 + 12x + 32, by the quantity x + 4:

x + 8

intercept form, what is the equation of the line passing through the point (2, 1) that is parallel to the line with the equation 3y + 12x = 6?

y =

intercept equation of the line with a slope of 4 and passing through the point (0, 15)?

y = 4x + 15 BONUS 10) MATH Short Answer Find the solution set for the following inequality: 15 - x ≥ 3x + 6 ANSWER: X ≤4/9

intercept equation of the line passing through the points (3, 8) and (2, 3)?

y = 5x - 7

18) MATH Short Answer Find the value of the following logarithmic expression: log25 5

½ (ACCEPT: 0.5) BONUS 18) MATH Short Answer What is the surface area of a closed box, in centimeters, with a width of 6, height of 4, and length of 10 centimeters, respectively? ANSWER: 248 (Solution: A = 2lw + 2lh + 2wh = 2(10)(6) + 2(10)(4) + 2(6)(4) = 248 cm)

23) MATH Short Answer Subtract the following, giving your answer in simplest radical form: ( squareroot20) − (2 squareroot45)

− 4 squareroot5

23) EARTH SCIENCE Short Answer Order the following 4 elements from the MOST common in the universe to the LEAST common: carbon; helium; hydrogen; iron

HYDROGEN; HELIUM; CARBON; IRON

1) PHYSICAL SCIENCE Short Answer If a solution has a low pH, what ion predominates in the solution?

HYDRONIUM (ACCEPT: HYDROGEN or H3O+ or H+)

22) LIFE SCIENCE Short Answer Consider a microbial species that lives in the gut of cows and breaks down cellulose. The cow benefits from the microbe predigesting cellulose and the microbe benefits from the cow by having a place to live. What is scientific term for this sort of symbiosis?

MUTUALISM (DO NOT ACCEPT: SYMBIOSIS OR COMMENSALISM)

5) GENERAL SCIENCE Short Answer Name all of the following 4 units of the international system of units that are considered base units and NOT considered derived: Newton, kilogram, second, hertz

KILOGRAM AND SECOND

13) PHYSICAL SCIENCE Short Answer Which one of the following 5 molecules is a disaccharide that is present in high levels in milk: maltose; fructose; glucose; lactose; hexose

LACTOSE

14) EARTH SCIENCE Short Answer Of the following 4 tectonic plates, name the LARGEST and the SMALLEST: the Pacific plate; the North American plate; the Juan de Fuca plate; the African plate

LARGEST = PACIFIC PLATE; SMALLEST = THE JUAN DE FUCA PLATE

8) LIFE SCIENCE Short Answer Name all of the following 4 organisms that molt: nematode; clam; lobster; mollusk

LOBSTER

22) EARTH SCIENCE Short Answer Order the following 3 stages of a stars development from the EARLIEST to the LATEST: Red Giant; white dwarf; main sequence star

MAIN SEQUENCE; RED GIANT; WHITE DWARF

24) MATH Short Answer What are the median, mean and mode, respectively, of the following set of 5 numbers, respectively: 35, 25, 10, 25, 45

MEDIAN = 25; MEAN = 28; MODE = 25

25) GENERAL SCIENCE Short Answer What are the SI units for length and time?

METER AND SECOND

6) LIFE SCIENCE Short Answer What process involves a nucleated cell doubling its DNA and duplicating chromosomes so the cell can then divide and produce two identical daughter cells?

MITOSIS

25) LIFE SCIENCE Short Answer What is the scientific term for the process arthropods go through to shed their exoskeleton to allow them to grow into a new one?

MOLTING

15) LIFE SCIENCE Short Answer Flowering plants are grouped into what 2 classes named after their number of seed leaves?

MONOCOTS AND DICOTS (ACCEPT: MONOCOTYLEDONS AND DICOTYLEDONS )

21) LIFE SCIENCE Short Answer What is the common name for the vector of Eastern Equine Encephalitis and malaria?

MOSQUITO

16) PHYSICAL SCIENCE Short Answer Of the following 3 forms of radioactive decay, which one is the MOST penetrating in human tissue and which one is the LEAST penetrating, respectively: alpha; beta; gamma

MOST = GAMMA; LEAST = ALPHA

3) MATH Short Answer What are the multiplicative inverse and additive inverse of -5/3 , respectively?

MULTIPLICATIVE INVERSE: -3/5, ADDITIVE INVERSE: 5/3

10) LIFE SCIENCE Short Answer Of the 4 main blood types, what type has the LEAST amount of reacting antigens and is therefore the most common type for blood transfusions?

O

19) EARTH SCIENCE Short Answer The bulk of the thermal energy at the Earth's surface is stored in what major reservoir?

OCEANS (ACCEPT: SEAS) (DO NOT ACCEPT: WATER)

14) EARTH SCIENCE Short Answer What is the name for the highest mountain on any planet in the Solar System?

OLYMPUS (ACCEPT: OLYMPUS MONS OR MT. OLYMPUS)

14) PHYSICAL SCIENCE Short Answer Rusting of iron is a chemical combination of what two chemical elements?

OXYGEN AND IRON

15) EARTH SCIENCE Short Answer What is the name for the gas found primarily in the stratosphere that absorbs UV radiation and ultimately protects life on Earth?

OZONE

10) LIFE SCIENCE Short Answer In the wall of the human heart's right atrium is an area which sets the rate of heart beats. What is the common name for this area?

PACEMAKER

22) LIFE SCIENCE Short Answer Which one of the following 5 trees would be LEAST characteristic in a temperate deciduous forest: maple; aspen; red oak; birch; palm

PALM

21) LIFE SCIENCE Short Answer What symbiotic relationship is characterized by one organism benefiting while the other is being harmed?

PARASITISM

16) LIFE SCIENCE Short Answer What is the scientific name for the basic cell type in higher plants that is relatively unspecialized, has a thin cell wall, and carries out most of the metabolism of the plant?

PARENCHYMA

4) EARTH SCIENCE Short Answer Eras on the Geological Time Scale are directly subdivided into what time units?

PERIODS

23) LIFE SCIENCE Short Answer What is the scientific term for the stalk of a leaf that joins the leaf to the node of a plant stem?

PETIOLE

25) LIFE SCIENCE Short Answer Of the four general types of biological molecules, proteins, carbohydrates, lipids and nucleic acids, under which one would insulin be categorized?

PROTEINS

10) LIFE SCIENCE Short Answer What is the scientific name for the veins in humans that return oxygen rich blood to the heart?

PULMONARY

16) GENERAL SCIENCE Short Answer Of the following 5 choices, which is the poorest conductor of electricity: sulfuric acid solution; copper wire; pure aluminum; salt solution; pure water

PURE WATER

fights): ringworm; jock itch; athletes foot

RINGWORM; JOCK ITCH; ATHLETES FOOT (ACCEPT: ALL OF THEM)

colored eyes in humans in some photographs taken with a flash: W) light that reflects off the retina X) light of a certain wavelength that is scattered by the cornea Y) light that is illuminating the optic nerve Z) certain films react with red light more than others

W) LIGHT THAT REFLECTS OFF THE RETINA

3) EARTH SCIENCE Multiple Choice As the eyewall of hurricane Katrina approached Louisiana, on which of the following sides of the hurricane were the highest speeds clocked: W) north X) south Y) east Z) west

W) NORTH

waves

W) P

10) GENERAL SCIENCE Multiple Choice Which of the following BEST describes the composition of the Proton Exchange Membrane in fuel cells used to power automobiles: W) polymer plastic X) metal foil Y) organic fiber Z) biofilm

W) POLYMER PLASTIC

bonded metals

W) POLYMERS

PHYSICAL SCIENCE Multiple Choice Which of the following is NOT a salt: W) potassium hydroxide X) potassium bromide Y) magnesium chloride Z) calcium carbonate

W) POTASSIUM HYDROXIDE

25) EARTH SCIENCE Short Answer Giving your answer as north, south, east, or west, plate tectonics show that the eastern side of the North American Plate is moving slowly in what general direction?

WEST

6) PHYSICAL SCIENCE Short Answer If a screwdriver is used to tighten a screw, then the screwdriver is an example of what simple machine?

WHEEL AND AXLE

2) PHYSICAL SCIENCE Short Answer What color of light is produced when equal intensities of green and magenta light are combined?

WHITE (Solution: they are complementary colors)

13) PHYSICAL SCIENCE Multiple Choice Which of the following must increase for the particles in a closed container of a constant volume if the temperature in the container increases: W) kinetic energy X) heat of fusion Y) total mass Z) mechanical energy

W) KINETIC ENERGY

7) MATH Short Answer Solve the following inequality for x: !2x - 3! < 9 (read as: open absolute value, 2x minus 3, close absolute value, is less than 9)

-3 < x < 6 (ACCEPT: X IS GREATER THAN -3 AND LESS THAN 6)

8) MATH Short Answer Evaluate the following radical: - squareroot (144/9)

-4

4) MATH Short Answer Evaluate the following radical: - 160,000

-400

20) MATH Short Answer Expand the following expression, giving your answer in standard binomial form: (4x + 3y) 2

16x 2 + 24xy + 9y 2

11) MATH Short Answer Expand the following expression, giving your answer in standard binomial form: (4x 4 y 2 + 3)2

16x 8 y 4 + 24x 4 y 2 + 9

10) MATH Short Answer Solve the following radical equation for x, when x is a real number: ( 2) 2 4 x ! =

18

7) LIFE SCIENCE Short Answer Stomata are each made of how many guard cells?

2

23) MATH Short Answer If Dan can paint a wall in 3 hours but Ryan takes 6 hours, how many hours will it take them to paint the wall if they work together?

2 (Solution: 1/x = 1/3 + 1/6; x = 2hrs)

6) PHYSICAL SCIENCE Short Answer Considering that the atomic mass of sodium is 23 and the atomic mass of chlorine is 35, what is the molarity of a solution where 116 grams of sodium chloride are added to 1,000 milliliters of water?

2 (Solution: 35 + 23 ! 116 = 2 moles of NaCl in 1 liter = 2 molar)

11) PHYSICAL SCIENCE Short Answer An alpha particle is made of how many protons and how many neutrons?

2 AND 2

24) EARTH SCIENCE Short Answer If the Earth were a perfect sphere without large continents and land masses, how many low and high equally proportioned tides would occur each lunar day?

2 HIGH AND 2 LOW

23) PHYSICAL SCIENCE Short Answer What is the weight per volume percent concentration of a solution made by mixing 20 grams of glucose with enough water to make 1 liter of solution?

2% (Solution: w/v% = 20g/1000ml × 100 = 2%)

16) MATH Short Answer If a fair die is tossed once, what is the probability that an odd number or a number less than 3 is thrown expressed as a fraction in lowest terms?

2/3

24) MATH Short Answer Evaluate the following expression: !20 - 30! + !30 - 20! (read as: open absolute value 20 minus 30 close absolute value, plus open absolute value 30 minus 20, close absolute value)

20

9) MATH Short Answer What is the percentage decrease when a stock price drops from $55.00 per share to $44.00 per share?

20%

3) MATH Short Answer In how many different ways can a team of 6 students be selected from a group of 10 students?

210 (Solution: nCr = n!/[r!(n

5) MATH Short Answer Evaluate the following expression: 6 + 2(5 + 4 - 1) (read as: 6 + 2, open parenthesis, 5 plus 4 minus 1, close parenthesis)

22

5) LIFE SCIENCE Short Answer Meiosis is typically divided into meiosis 1 and meiosis 2. In a typical human egg cell, how many chromosomes will it have in each cell after meiosis 1?

23

19) MATH Short Answer Name all the prime numbers greater than 20 and less than 30:

23 AND 29

21) MATH Short Answer Multiply the following 2 radicals, giving your answer to the nearest whole number: (3 4)(2 16) 3 3

24 (Solution: 3×2× 4 16 6 64 24 3 3 3 ! = = )

14) MATH Short Answer Giving your answer in terms of pi, what is the volume of a cone with a height of 8 meters and a base whose diameter is 6 meters?

24π (Solution: V = ⅓πr 2 h = ⅓(π)(32 )(8) = 24π)

13) MATH Short Answer Give the numerical value of 25 factorial divided by 24 factorial:

25

21) PHYSICAL SCIENCE Short Answer Assuming a planet has a gravitational force of 5 meters per second squared, what would an object's weight be, in newtons, if it has a specific gravity of 2 and a mass of 5 kilograms?

25

10) MATH Short Answer If it takes a hiker 2 hours to make a descent from 10,500 feet to 7,500 feet, what is his average rate of decent, in feet per minute?

25 (Solution: (10,500 - 7,500)/(120 - 0) = 25 ft/min)

20) MATH Short Answer Find the area of a triangle, in meters squared, given a base of 10 meters and height of 5 meters:

25 (Solution: A = ½(b)(h) = ½ (10)(5) = 25 m^2)

9) GENERAL SCIENCE Short Answer Giving your answer in meters rounded to the second decimal place, add 2,500 centimeters to 350 millimeters:

25.35 (Solution: 2500cm = 25m; 350mm = 0.35m; 25m + 0.35m = 25.35 meters)

12) PHYSICAL SCIENCE Short Answer Consider a 2.0 kilogram rock that falls vertically from a ledge and strikes the ground with a velocity of 50 meters per second. If friction is ignored, what was the gravitational potential energy, in joules, of the rock when it was on the ledge in relation to the place where it struck the ground?

2500 (Solution: ½mv 2 = ½(2)(50)2 = ½ × 2 × 2500 = 2500 J)

4) MATH Short Answer Find the lateral surface area, in square meters, of a right circular cylinder with radius 4 meters and height 10 meters. Use π = 3.14 and give your answer rounded to the nearest whole number:

251 (Solution: LA = 2πrh; = 2(3.14)(4)(10) = 251.2 m2)

24) GENERAL SCIENCE Short Answer At standard temperature and pressure, how many grams, rounded to the nearest whole number, does the contents in a 12 ounce or 355 milliliter water bottle weigh?

355

8) MATH Short Answer If on average 6 people drink two-thirds of a gallon of lemonade, how many people will drink 4 gallons?

36

7) LIFE SCIENCE Short Answer What would be the number of chromosomes in a normal human skin cell that went through one complete sequence of mitosis and did not go through cytokinesis?

92

20) LIFE SCIENCE Short Answer During early metaphase of mitosis, how many chromatids will be present in a single human cell?

92 (Solution: 2 chromatids/chromosome ! 46 x 2 = 92)

13) EARTH SCIENCE Multiple Choice Which of the following does NOT satisfy the scientific definition of a mineral: W) it is always an inorganic substance X) it has a distinct chemical composition Y) it can be a solid or a molten substance Z) it must form by natural processes

Y) IT CAN BE A SOLID OR A MOLTEN SUBSTANCE

viral drugs Z) the ticks that carry the disease are about the size of a sesame seed

Y) LYME DISEASE IS TREATED WITH ANTI

an) tube Z) siphon

Y) MALPIGHIAN TUBE

4) PHYSICAL SCIENCE Multiple Choice Which of the following statements is LEAST accurate regarding states of matter: W) solids when heated typically expand X) gases can be made of a single element Y) matter on Earth is never in a plasma state Z) gases condense to form liquids

Y) MATTER ON EARTH IS NEVER IN A PLASMA STATE

ocean ridge Z) continental shelf

Y) MID

13) EARTH SCIENCE Multiple Choice Which of the following is when ocean tidal currents typically reach their maximum velocity: W) at high tide extremes X) at low tide extremes Y) midway between high and low tides Z) during flood tide

Y) MIDWAY BETWEEN HIGH AND LOW TIDES

12) LIFE SCIENCE Multiple Choice Which of the following is NOT true about the human eye: W) the cornea acts as a lens with a fixed shape X) images that are formed on the retina are inverted Y) most of the eye is filled with a liquid called the aqueous humor Z) vitamin A is important for light absorbing molecules critical in vision

Y) MOST OF THE EYE IS FILLED WITH A LIQUID CALLED THE AQUEOUS HUMOR

6) GENERAL SCIENCE Multiple Choice The U.S. contains a vast energy resource that could fuel the nation for several hundred more years at our current rate of consumption if it were not so difficult and expensive to mine. Which of the following BEST describes this resource: W) tar sands X) deep underground peat beds Y) oil shale Z) propane

Y) OIL SHALE

21) PHYSICAL SCIENCE Multiple Choice Which of the following is LEAST accurate: W) all atoms have protons in their nuclei X) covalent bonding of atoms involves the sharing of electrons Y) potassium and bromine normally form covalent bonds Z) most of the mass of an atom is in its nucleus and the atom is mostly empty space

Y) POTASSIUM AND BROMINE NORMALLY FORM COVALENT BONDS

12) GENERAL SCIENCE Multiple Choice A dog has a difficult time locating a stick at the bottom of a clear, shallow pool because he perceives it to be in a slightly different spot. This is BEST explained by which of the following properties of light: W) diffraction X) reflection Y) refraction Z) polarization

Y) REFRACTION

1) LIFE SCIENCE Multiple Choice What is the name for the basic five carbon sugar that is present in slightly different forms in both DNA and RNA nucleotides: W) glucose X) fructose Y) ribose Z) maltose

Y) RIBOSE

Saharan Africa and characterized by the dominant plants being fire resistant: W) chaparral X) tundra Y) savanna Z) temperate

Y) SAVANNA

20) GENERAL SCIENCE Short Answer Consider the following ecosystem and name the secondary consumer: A plant that derives energy from the Sun is eaten by a rabbit; the rabbit is eaten by a wolf; the scraps of rabbit are eaten by a hawk; and the remaining carcass is left to decompose

WOLF

7) GENERAL SCIENCE Multiple Choice Which of the following is NOT true of metric volumes: W) a liter is larger than a centiliter X) a milliliter is smaller than a picoliter Y) a deciliter is smaller than a liter Z) a microliter is smaller than a milliliter

X) A MILLILITER IS SMALLER THAN A PICOLITER

11) PHYSICAL SCIENCE Multiple Choice Which of the following is the BEST description of a colloid: W) a homogeneous mixture of uniform particles in solution X) a mixture of different substances that never settle out of solution Y) ionized particles in suspension Z) unfilterable suspended solutions

X) A MIXTURE OF DIFFERENT SUBSTANCES THAT NEVER SETTLE OUT OF SOLUTION

21) GENERAL SCIENCE Multiple Choice Which of the following materials is the STRONGEST in terms of tensile strength per mass: W) steel X) a spider's web Y) cotton fibers Z) nylon fibers

X) A SPIDER'S WEB

3) EARTH SCIENCE Multiple Choice Which of the following is the primary reason for the decline in fish populations in New England lakes, rivers and streams: W) destruction of native species by invasive species X) acid rain Y) fertilizer runoff Z) over fishing

X) ACID RAIN

3) GENERAL SCIENCE Multiple Choice Which of the following forms of coal typically has the HIGHEST BTU value per pound: W) lignite X) anthracite Y) bituminous Z) peat

X) ANTHRACITE

6) GENERAL SCIENCE Multiple Choice Which of the following forms of coal typically has the HIGHEST percent carbon content: W) peat X) anthracite Y) bituminous Z) lignite

X) ANTHRACITE

20) LIFE SCIENCE Multiple Choice Which of the following is the phylum to which organisms with large amounts of chitin belong: W) Anthophyta X) Arthropoda Y) Vertebrata Z) Ginkophyta

X) ARTHROPODA

tech cements

X) AS AN ABRASIVE

13) EARTH SCIENCE Multiple Choice Which of the following is a layer of the Earth that is primarily characterized as rock with the properties of being hot, weak and plastic: W) lithosphere X) asthenosphere Y) inner core Z) outer core

X) ASTHENOSPHERE

12) GENERAL SCIENCE Multiple Choice Which of the following scientists would most likely use a coronagraph in their experiments: W) biologist X) astronomer Y) paleontologist Z) oceanographer

X) ASTRONOMER

22) EARTH SCIENCE Multiple Choice Oceanic crust being formed at spreading centers is primarily made of: W) granite X) basalt Y) peridotite Z) andesite

X) BASALT

7) LIFE SCIENCE Multiple Choice Which of the following is the BEST description for the location in the human body of the esophagus: W) behind the trachea connecting the mouth to the lungs X) behind the trachea connecting the mouth to the stomach Y) in front of the trachea connecting the mouth to the stomach Z) in front of the trachea connecting the mouth with the lungs

X) BEHIND THE TRACHEA CONNECTING THE MOUTH TO THE STOMACH

18) EARTH SCIENCE Multiple Choice Calcite belongs to which of the following mineral groups: W) oxide X) carbonate Y) sulfate Z) silicate

X) CARBONATE

4) LIFE SCIENCE Multiple Choice Which of the following is the BEST example of how evolution progresses: W) animals learn to survive through experience and teach their offspring X) certain mutations improve an organism's offspring's chances to survive and reproduce in different environments Y) animals develop certain strengths and weaknesses through experience and pass the knowledge down by example Z) an animal that is successful at hunting will be more likely to survive

X) CERTAIN MUTATIONS IMPROVE AN ORGANISM'S OFFSPRING'S CHANCES TO SURVIVE AND REPRODUCE IN DIFFERENT ENVIRONMENTS

7) EARTH SCIENCE Multiple Choice Which of the following is a volcano that is generally the smallest type, has very steep slopes, and would most likely result from a single violent eruption: W) stratovolcano X) cinder cone Y) shield volcano Z) lava dome

X) CINDER CONE

17) EARTH SCIENCE Multiple Choice A conglomerate rock belongs to which of the following groups of rocks: W) foliated metamorphic X) clastic sedimentary Y) chemical sedimentary Z) organic sedimentary

X) CLASTIC SEDIMENTARY

7) EARTH SCIENCE Multiple Choice Which of the following is believed to be the process that causes Earth's tectonic plates to move: W) convection currents in the crust X) convection currents in the mantle Y) gravitational influence of the Sun and Moon directly on continental plates Z) volcanic activity along subduction zones

X) CONVECTION CURRENTS IN THE MANTLE

1) LIFE SCIENCE Multiple Choice Which of the following is a segmented worm: W) hook worm X) earthworm Y) round worm Z) nematode

X) EARTHWORM

13) PHYSICAL SCIENCE Multiple Choice If the work input remains the same and there is an increase in the output of a simple machine, there would also have to be an increase in the machine's: W) power X) efficiency Y) friction Z) effort force

X) EFFICIENCY

8) EARTH SCIENCE Multiple Choice Which of the following is the actual location where slippage of rock masses first occurs in an earthquake: W) seismic front X) focus Y) Richter point Z) foreshock

X) FOCUS

grade metamorphic rock: W) clastic X) foliated Y) intrusive Z) extrusive

X) FOLIATED

14) EARTH SCIENCE Multiple Choice Which of the following is a simple device that can be used to show that the Earth rotates on its axis: W) the astrolabe X) Foucault Pendulum Y) gnomon Z) an interferometer

X) FOUCAULT PENDULUM

6) GENERAL SCIENCE Multiple Choice Although bioflavinoids and phytochemicals are not true vitamins, they are commonly sold as a vitamin supplement and are most commonly known for their antioxidant activity. Which of the following is the BEST dietary source for these compounds: W) fish X) fruit Y) meat Z) dairy

X) FRUIT

11) EARTH SCIENCE Multiple Choice Which of the following is the BEST reason why the Earth's inner core is solid and the outer core is not: W) greater depths yield cooler temperatures X) greater depths yield higher pressures Y) inner core is further removed from the influence of hot magma Z) they have different mineral contents

X) GREATER DEPTHS YIELD HIGHER PRESSURES

meter) is used to measure: W) salinity X) humidity Y) wind direction Z) pressure

X) HUMIDITY

12) EARTH SCIENCE Multiple Choice Rocks that are produced when magma cools and solidifies deep below the Earth's crust are called: W) extrusive igneous X) intrusive igneous Y) extrusive metamorphic Z) intrusive sedimentary

X) INTRUSIVE IGNEOUS

5) GENERAL SCIENCE Multiple Choice Which of the following is the MOST accurate description of a Newtonian reflector telescope: W) it has one flat primary mirror and two ocular lenses X) it has a concave parabolic main or primary mirror, a flat secondary mirror, and an eyepiece lens Y) it has a primary glass lens, a secondary flat mirror, and an ocular lens Z) it has 2 convex glass lenses and an ocular lens

X) IT HAS A CONCAVE PARABOLIC MAIN OR PRIMARY MIRROR, A FLAT SECONDARY MIRROR, AND AN EYEPIECE LENS

8) EARTH SCIENCE Multiple Choice In a 24 hour period, when does the coldest temperature of the day typically occur: W) at midnight X) just after sunrise Y) just after sunset Z) between 3 and 4 AM

X) JUST AFTER SUNRISE

fiber reinforced mirrors

X) LINKING MULTIPLE TELESCOPES ON EARTH BY COMPUTERS

17) EARTH SCIENCE Multiple Choice There are 4 new statues in a park in New York city. The statue composed of which of the following materials will most likely deteriorate by chemical weathering the quickest: W) granite X) marble Y) slate Z) schist

X) MARBLE

11) GENERAL SCIENCE Multiple Choice Which of the following types of scientists would MOST likely use agar in their experiments: W) biochemist X) microbiologist Y) geologist Z) physicist

X) MICROBIOLOGIST

17) PHYSICAL SCIENCE Multiple Choice Air is best described as a: W) compound X) mixture Y) solution Z) hydrocarbon

X) MIXTURE

19) EARTH SCIENCE Multiple Choice Which of the following is NOT true about glaciations: W) the land under the West Antarctic Ice Sheet is being pushed down below sea level by the weight of the ice above X) most of the icebergs found in the North Atlantic come from the Iceland coast Y) current ice sheets can be nearly 2 miles thick Z) glaciers can form near the equator at high altitude

X) MOST OF THE ICEBERGS FOUND IN THE NORTH ATLANTIC COME FROM THE ICELAND COAST (Solution: they come from Greenland)

22) GENERAL SCIENCE Multiple Choice Which of the following is NOT typically considered an alternative energy source: W) hot springs X) natural gas Y) wind energy Z) sun energy

X) NATURAL GAS

25) LIFE SCIENCE Multiple Choice Which of the following characteristics is MOST typical of bacteria: W) numerous mitochondria X) no nucleus Y) numerous cilia Z) reproduction by budding

X) NO NUCLEUS

6) PHYSICAL SCIENCE Multiple Choice Which of the following atomic properties most directly determines that elements in the same group of the Periodic Table have similar chemical properties: W) number of protons X) number of valence electrons Y) charge of protons Z) stability of neutrons

X) NUMBER OF VALENCE ELECTRONS

8) PHYSICAL SCIENCE Multiple Choice A carboxyl group is the functional group in which of the following classes of compounds: W) ester X) organic acid Y) halocarbon Z) hydrocarbon

X) ORGANIC ACID

18) PHYSICAL SCIENCE Multiple Choice Which of the following is the BEST evidence that light travels as a wave: W) a metal that is heated produces different colors X) passing light through two pinholes can produce an interference pattern Y) light has a constant speed in a vacuum Z) high frequencies have short wavelengths

X) PASSING LIGHT THROUGH TWO PINHOLES CAN PRODUCE AN INTERFERENCE PATTERN

8) PHYSICAL SCIENCE Multiple Choice Acids are often defined as proton donors. Which of the following is the common form of this proton: W) salts X) positive hydrogen ion Y) positron Z) buffer

X) POSITIVE HYDROGEN ION

9) GENERAL SCIENCE Multiple Choice Which of the following elements is essential to the human body for muscle and nerve function and maintaining fluid balance: W) iodine X) potassium Y) calcium Z) iron

X) POTASSIUM

17) EARTH SCIENCE Multiple Choice Where does most of Earth's groundwater come from: W) percolation of seawater into underground aquifers where it undergoes natural desalination X) precipitation Y) aquifers that have been there since Earth's origin Z) glacial melting and runoff

X) PRECIPITATION

10) EARTH SCIENCE Multiple Choice An organism that inhabits a fluvial environment is one that inhabits: W) lakes and ponds X) rivers and streams Y) deserts Z) volcanoes

X) RIVERS AND STREAMS

sized herbivore Y) Brachiosaurus had tail spikes and bony plates along the length of its back Z) Tyrannosaurus rex was a large carnivorous dinosaur

Y) BRACHIOSAURUS HAD TAIL SPIKES AND BONY PLATES ALONG THE LENGTH OF ITS BACK

12) PHYSICAL SCIENCE Multiple Choice Which of the following elements is NOT a gas in its standard state: W) fluorine X) nitrogen Y) bromine Z) neon

Y) BROMINE

1) PHYSICAL SCIENCE Multiple Choice Which of the following statements is TRUE: W) for a heavy element to be stable the number of protons typically equals the number of neutrons X) the faster a boiling uncovered pot of water boils the hotter the water becomes Y) cellulose is an example of a natural polymer Z) the speed of light is always the same

Y) CELLULOSE IS AN EXAMPLE OF A NATURAL POLYMER

6) PHYSICAL SCIENCE Multiple Choice Which of the following statements is TRUE of buffers: W) all neutral pH solutions have equal numbers of buffers X) hydronium ions are present in large amounts in most buffers Y) certain buffers will absorb hydrogen ions to prevent a change in a solution's pH Z) buffers are artificial chemicals rarely found in nature

Y) CERTAIN BUFFERS WILL ABSORB HYDROGEN IONS TO PREVENT A CHANGE IN A SOLUTION'S PH

5) EARTH SCIENCE Multiple Choice Except for birds, dinosaurs became extinct after which of the following periods: W) Jurassic X) Triassic Y) Cretaceous Z) Permian

Y) CRETACEOUS

sex cells have the same chromosomes but appear and function in greatly different ways: W) different cells lose and acquire different genes as they mature X) as cells mature they develop different genes Y) different genes are turned on and off as cells mature Z) cells transport pieces of chromosomes called plasmids to control how different cells will function and appear

Y) DIFFERENT GENES ARE TURNED ON AND OFF AS CELLS MATURE

11) PHYSICAL SCIENCE Multiple Choice Which of the following is LEAST accurate: W) water is a molecule composed of two hydrogen atoms and one oxygen atom X) each carbon dioxide molecule has 2 oxygen atoms Y) dioxide means something has two oxygen molecules Z) carbon dioxide is made of two elements

Y) DIOXIDE MEANS SOMETHING HAS TWO OXYGEN MOLECULES

1) LIFE SCIENCE Multiple Choice Which of the following is the MOST common manner for transmission of beef tapeworms to humans: W) drinking contaminated water X) swimming in contaminated water Y) eating uncooked beef Z) stepping in cow manure

Y) EATING UNCOOKED BEEF

12) PHYSICAL SCIENCE Multiple Choice Which of the following is an example of laboratory equipment that is specifically designed to mix fluids without spilling: W) pipette X) centrifuge Y) Erlenmeyer flask Z) graduated cylinder

Y) ERLENMEYER FLASK

4) EARTH SCIENCE Multiple Choice Sinuous ridges of sand and gravel that form beneath the lower end of a glacier in the ablation zone are called: W) sandbars X) fjords Y) eskers Z) dikes

Y) ESKERS

1) GENERAL SCIENCE Multiple Choice Which of the following types of scientists would most likely use the PCR, or polymerase chain reaction, method in their experiments: W) nuclear chemist X) astronomer Y) geneticist Z) physicist

Y) GENETICIST

like appearance Y) have large crystals Z) have a soft outer layer and a dense hard inner core

Y) HAVE LARGE CRYSTALS

16) GENERAL SCIENCE Multiple Choice Which of the following diseases is LEAST likely to respond to antibiotics: W) pertussis or whooping cough X) tetanus Y) hepatitis C Z) strep throat

Y) HEPATITIS C

14) GENERAL SCIENCE Multiple Choice Which of the following BEST describes the position of a waxing nearly half Moon just after an April sunset in Mississippi: W) low in the east X) low in the west Y) high in the southern sky Z) high in the northern sky

Y) HIGH IN THE SOUTHERN SKY

16) GENERAL SCIENCE Multiple Choice Which of the following is NOT true: W) when diluting strong acids, it is safer to pour the acid into the water X) carbon monoxide is a poisonous gas Y) iron plays a central role in the chlorophyll molecule Z) the pH of soapy water is basic

Y) IRON PLAYS A CENTRAL ROLE IN THE CHLOROPHYLL MOLECULE (Solution: magnesium in chlorophyll, iron in hemoglobin)

18) MATH Short Answer What is the sum of all odd one-digit prime positive numbers?

15

chain hydrocarbon

Z) LONG

13) MATH Short Answer Solve the following logarithmic equation for x: logx 25 1 = -2

5

16) PHYSICAL SCIENCE Short Answer What is the total pressure, in atmospheres, at a depth of 40 meters in saltwater?

5 (ACCEPT 4.87)

7) MATH Short Answer Add the following radicals, giving your answer in simplest radical form: 3squareroot 3 + squareroot12

5 squareroot3

6) MATH Short Answer Giving your answer as a proper fraction in its simplest form, multiply the following two fractions: (-5/6)*(-6/11)

5/11

21) EARTH SCIENCE Short Answer A third quarter moon has what percent of its surface appearing to be illuminated to an observer on Earth?

50

8) MATH Short Answer Consider a smaller circle drawn within a larger circle. If the diameter of the larger circle is 10 centimeters and the diameter of the smaller circle is 6 centimeters, what is the area of the space between the two circles, giving your answer in square centimeters to the nearest whole number?

50

hertz wave, in cycles per second, if the wave has an amplitude of 2 meters?

50

1) LIFE SCIENCE Short Answer Tommy has blue eyes and his wife Jean has brown eyes. Tommy's father has brown eyes and Jean's father has blue eyes. What percentage of Tommy and Jean's children would be expected to have blue eyes, assuming blue eyes are recessive?

50%

6) LIFE SCIENCE Short Answer Arachnids have how many pairs of appendages?

6

14) MATH Short Answer What is the degree of the following polynomial: 7x 4 y -5x 3 y 3 + 3xy 3 - 4

6 (Solution: the degree of the term with the largest degree, 3 + 3)

24) GENERAL SCIENCE Short Answer The time aboard a ship at sea is 12 noon. If at the same time, the time at the Prime Meridian time is 4 PM, what is the ship's most likely longitude, in degrees and as east or west?

60º WEST

25) MATH Short Answer What three consecutive numbers add up to 192?

63, 64, 65

kilogram person towards the Earth's surface?

686 (ACCEPT: 700)

18) PHYSICAL SCIENCE Short Answer If sodium has an atomic mass of 23 and an atomic number of 11, what is the mass, in grams, of 3 moles of sodium?

69

10x^2 )

6x^3

1) PHYSICAL SCIENCE Short Answer What is the pH of a neutral aqueous solution?

7

3) PHYSICAL SCIENCE Short Answer If a block of wood floats with 22% of its volume not submerged, what is its density, to the second decimal place in grams per cubic centimeter?

0.78

21) PHYSICAL SCIENCE Short Answer How many protons does the heaviest isotope of hydrogen have?

1

23) LIFE SCIENCE Short Answer How many cotyledons are found in seedlings of plants that develop into adult plants with parallel veins in their leaves and flower parts usually in multiples of three?

1

3) PHYSICAL SCIENCE Short Answer What is the mass, in grams, of 1 cubic centimeter of water at standard temperature and pressure?

1

25) MATH Short Answer Simplify the following, giving your answer in standard form: (x+ x^2 )/x

1 + x (ACCEPT: x + 1)

1) PHYSICAL SCIENCE Short Answer What is the mass, in grams, of a 1 kilogram object on a planet with a gravitational force of 3 meters per second squared?

1,000

4) MATH Short Answer Arrange the following 4 fractions from the LARGEST to the SMALLEST: 13/18, 7/12, 10/4, 2/12

10/4, 13/18, 7/12, 2/12

24) GENERAL SCIENCE Short Answer Convert 100 milliliters into cubic centimeters:

100

25) GENERAL SCIENCE Short Answer 10 kilowatts is equal to how many watts?

10000

17) MATH Short Answer Give the third term in the binomial expansion of the following: (x + y) 5

10x 3 y 2

19) MATH Short Answer Rounded to the nearest meter, if side A of a right triangle measures 6 meters and side B measures 9 meters, what is the length of the hypotenuse?

11

9) PHYSICAL SCIENCE Short Answer How many grams are in 2 moles of sodium chloride, knowing that sodium chloride has a molecular weight of 58?

116 (Solution: 58g/mole × 2 moles = 116 grams)

16) MATH Short Answer If the time is 1:15 PM, what time will it be, given as hours, minutes and AM or PM, in 630 minutes?

11:45 PM

1) GENERAL SCIENCE Short Answer Giving your answer in liters and rounded to the first decimal place, what volume of fluid results after adding 3,400 milliliters of solution A and 34 deciliters of solution B to 5.2 liters of water?

12.0

14) GENERAL SCIENCE Short Answer Multiply the following 2 values, giving your answer in the correct number of significant figures: 10.1 × 1.201

12.1

letter arrangements are there of the first 6 letters of the alphabet?

120 (Solution: P(6,3) = 6!/(6

4) MATH Short Answer What is the average of the positive whole numbers of 1 through 29?

15

9) PHYSICAL SCIENCE Short Answer Consider an atom that has an original atomic mass of 33 and has 17 neutrons. If it then loses one proton, what is the resultant atomic number of the atom?

15 (Solution: atomic mass - 17 N = 16 protons = atomic # of 16 ! loss of 1 is 16 - 1 = 15)

9) MATH Short Answer Giving your answer to the third decimal place, subtract the following two numbers: 18 - 2.049

15.951

14) MATH Short Answer What is the surface area of a cube, in square feet, if one edge measures 5 feet?

150

18) MATH Short Answer Solve the following equation for x over the real number: (64)2 = 16x 2

16 and

24) MATH Short Answer Giving your answer as a decimal to two places, divide the following: 11/327

29.73

25) MATH Short Answer Simplify the following expression if x represents a negative quantity: 4 1 4 ! (16x )

2x

18) LIFE SCIENCE Short Answer How many nucleotide bases make up an anticodon?

3

21) MATH Short Answer If side A of a right triangle measures 4 meters and the hypotenuse measures 5 meters, what is the length of side B, in meters?

3 (Solution: 3:4:5 triangle)

11) MATH Short Answer Remove all negative exponents and simplify the following expression: x x y z 4 4 0 −3 4

3 4 xy z (ACCEPT: y x z 3 4 )

21) MATH Short Answer Order the following 4 radicals from the smallest to the largest: 3 2 ; 3 2 ; 3 ; 2

3 2 ; 2 ; 3 ; 3 2

17) GENERAL SCIENCE Short Answer Giving your answer in angstroms, add 2 nanometers to 10 angstroms:

30 (Solution: 2 nm = 20A; 20A + 10A = 30A)

23) PHYSICAL SCIENCE Short Answer How many amperes of current are running through a 3,600 watt space heater on a 120 volt electric line?

30 (Solution: current = watts/volts; 3600/120 = 30 amperes)

6) MATH Short Answer Giving your answer in terms of pi, what is the volume, in cubic meters, of a right circular cone that has a height of 10 meters and a radius of 3 meters?

30π

13) MATH Short Answer Using π = 3.14, give the integer value, in square centimeters, for the area of a circle whose diameter is 200 centimeters:

31,400 (Solution: A = πr 2 = (3.14)(100)2 = 31,400 cm2)

mile trip would be represented by how many centimeters on the same map?

32

2) GENERAL SCIENCE Short Answer To the nearest whole number, convert 53ºC into kelvin:

326

9) MATH Short Answer Express the following in its simplest radical form: (5 + 2 )(7 - 2 )

33 + 2 2 (ACCEPT: 2 2 + 33)

23) MATH Short Answer Name all of the following 5 numbers that are composite numbers: 3, 11, 33, 53, 63

33; 63

1) GENERAL SCIENCE Short Answer Giving your answer in grams, add 3000 centigrams to 5000 milligrams:

35 (Solution: 3000 cg = 30g; 5000 mg = 5g; 30g + 5g = 35 grams)

kilogram iron block across a horizontal surface if 700 newtons is applied to the block?

35 (Solution: A = F/M; A = 700/20; A = 35 m/sec^2)

10) MATH Short Answer If the surface area of a right rectangular prism is 188 cubic centimeters and its length is 6 centimeters and its height is 7 centimeters, find its width, to the nearest whole centimeter:

4

3) MATH Short Answer Consider 2 cars that start from the same point along a highway and travel in the same direction. One car travels at a constant rate of 40 miles per hour and the other at a constant rate of 50 miles per hour. How many hours later are the cars 40 miles apart?

4

5) LIFE SCIENCE Short Answer How many canine teeth do humans typically have in early adulthood?

4

quarter that needed to lift a resistance force?

4

6) MATH Short Answer Express the following radical in its simplest rationalized form: 8 3

4 3 2

5) MATH Short Answer How many 4 ounce containers can be packed from 1200 pounds of dry tea leaves?

4,800 (Solution: 4 × 1200 = 4800)

6) GENERAL SCIENCE Short Answer Giving your answer in grams, add 2,000 milligrams to 0.04 kilograms:

42 (Solution: 2000 mg = 2g; 0.04kg = 40g; 2g + 40g = 42 grams)

15) MATH Short Answer Consider an isosceles triangle whose 2 sides of equal length meet to form a vertex angle of 96º. What is the degree measure of each base angle?

42 (Solution: 2x + 96 = 180; x = 42º)

3) MATH Short Answer Find the sum of all interior angles, in degrees, in a regular hexagon:

720 (Solution: (6 - 2)(180) = 720º)

13) PHYSICAL SCIENCE Short Answer If 50 grams of one reactant chemically combines with 25 grams of another reactant to form a product, what is the total mass, in grams, of the product?

75

13) MATH Short Answer Find the surface area, in terms of pi in inches squared, of a closed right circular cylinder given that its radius is 3 inches and its length or height is 10 inches:

78π (Solution: SA = 2πr 2 + 2πrh; SA = 2(π)(32 ) + 2(π)(3)(10) = 78π)

23) PHYSICAL SCIENCE Short Answer What is the ideal mechanical advantage of a ramp 16 meters long and 2 meters high that has a frictional coefficient of 2?

8

7) GENERAL SCIENCE Short Answer Giving your answer in proper scientific notation with the correct number of significant digits, convert 800 kilometers per hour into meters per hour:

8 × 10^5

7) PHYSICAL SCIENCE Short Answer If a force of 200 newtons is required to accelerate an object at 25 meters per second squared on a horizontal frictionless surface, what is the mass of the object, in grams?

8,000 (Solution: F/a = m = 200/25 = 8 kg = 8,000 g)

19) PHYSICAL SCIENCE Short Answer A chemistry experiment calls for 10.0 liters of a 0.8 molar solution of a chemical with a molecular weight of 100 grams per mole. How many grams of the chemical will you need to weigh out to make this solution?

800 (Solution: 10L × 0.8 moles/L = 8 moles; 8 moles × 100 g/mole = 800 grams)

14) MATH Short Answer What is the degree measure of the vertex angle in an isosceles triangle if each base angle measures 48º?

84

5) MATH Short Answer If the volume of a rectangular pyramid is 36 cubic meters and its base measures 3 meters in length and 4 meters in width, find its height, in meters:

9 (Solution: V = 1/3(l × w × h); 36 = 1/3(3 × 4 × h); h = 9 m)

21) PHYSICAL SCIENCE Short Answer What is the mass, in grams, of a bag of beans that weighs 90 newtons if the force of gravity is assumed to be 10 meters per second squared?

9,000 (Solution: 90/10 × 1000 = 9000)

hunting of sea otters on the northwest coast led to an overabundance of sea urchins which in turn overgrazed on sea kelp almost wiping out the kelp forests. What is the scientific ecological term for what the sea otter is considered in this case as a central organism to this community?

A KEYSTONE SPECIES

12) LIFE SCIENCE Short Answer What is the general name for the twenty or so different building blocks from which most proteins are made?

AMINO ACIDS

6) PHYSICAL SCIENCE Short Answer What is the atomic mass rounded to the nearest whole number and the net charge of an ion with 7 protons, 6 neutrons, and 8 electrons?

ATOMIC MASS = 13; NET CHARGE = -1

25) GENERAL SCIENCE Short Answer Order the following 4 celestial objects from the LEAST massive to the MOST massive: neutron star; brown dwarf; the Sun; a galaxy

BROWN DWARF; SUN; NEUTRON STAR; GALAXY

20) GENERAL SCIENCE Short Answer Most plastics are based on monomers that are composed of what element that forms the backbone of the polymer?

CARBON

21) LIFE SCIENCE Short Answer What molecule is released from muscles as a waste product, transported to the lungs in blood, and exhaled as a gas?

CARBON DIOXIDE (ACCEPT: CO2)

24) GENERAL SCIENCE Short Answer If solid dry ice is placed in a sealed sandwich bag at room temperature, the bag quickly expands because of the release of what gas?

CARBON DIOXIDE (ACCEPT: CO2)

induced global climate change is believed to be caused primarily by the release of what gas into the atmosphere?

CARBON DIOXIDE (ACCEPT: CO2)

16) LIFE SCIENCE Short Answer What outer structure possessed by plant cells protects them from environmental changes and helps them maintain their shape during stress and times of water loss?

CELL WALL

17) LIFE SCIENCE Short Answer What structure is exterior to the plasma membrane of a plant cell that is absent from an animal cell?

CELL WALL (ACCEPT: WALL)

12) LIFE SCIENCE Short Answer Order the following 4 human vertebral bone types from UPPERMOST to the LOWERMOST in the vertebral column: sacral; lumbar; cervical; thoracic

CERVICAL; THORACIC; LUMBAR; SACRAL

21) PHYSICAL SCIENCE Short Answer Name all of the following 3 chemical compounds that are substituted hydrocarbons: CH4; CO2; CH3Br

CH3Br

15) EARTH SCIENCE Short Answer Order the following 4 layers of the Sun from the INNERMOST to the OUTERMOST: core; radiative zone; corona; convective zone

CORE; RADIATIVE ZONE; CONVECTIVE ZONE; CORONA TOSS

12) LIFE SCIENCE Short Answer What is the term for the front transparent covering of the human eye through which light travels first?

CORNEA

rays; cosmic rays; infrared light; ultrasound waves

COSMIC RAYS AND ULTRASOUND WAVES

17) EARTH SCIENCE Short Answer The lithosphere is made up of the uppermost mantle and what other layer?

CRUST

22) PHYSICAL SCIENCE Short Answer What is the specific name for the type of solid where the atoms or molecules of the solid are arranged in a geometric repeating arrangement?

CRYSTAL

9) GENERAL SCIENCE Short Answer Which one of the following 5 materials has the smallest critical angle causing light entering it to be more likely to undergo total internal reflection: pure water; a near perfect vacuum; diamond; Lucite; 100% ethanol

DIAMOND

15) MATH Short Answer Find the distance between and give the coordinates of the midpoint between the following points: (12, 8) and (18, 8)

DISTANCE = 6; MIDPOINT = (15, 8) (Solution: d = ( ) ( ) 2 2 1 2 2 1 x ! x + y ! y ; ( ) ( ) 2 2 18 !12 + 8 ! 8 = 6); midpoint x = (x1 + x2)/2 = (12 + 18)/2 = 15 ; midpoint y = (y1 + y2)/2 = (8 + 8)/2 = 8)

24) PHYSICAL SCIENCE Short Answer Name all of the following 4 particles that cannot be deflected when passing through a magnetic field: photon; electron; neutron; proton

NEUTRON AND PHOTON

18) EARTH SCIENCE Short Answer During what phase of the Moon can a solar eclipse occur?

NEW MOON

12) PHYSICAL SCIENCE Short Answer What metric unit is most appropriate for weight?

NEWTON

21) GENERAL SCIENCE Short Answer Arrange the following 4 ecological systems in order of increasing or broad level of complexity: population; community; niche; biosphere

NICHE; POPULATION; COMMUNITY; BIOSPHERE

resistant alloys, and is used in making rechargeable batteries?

NICKEL (ACCEPT: Ni)

23) EARTH SCIENCE Short Answer What three gases make up over 99% of all dissolved gases in surface sea water?

NITROGEN, CARBON DIOXIDE AND OXYGEN (in any order)

18) GENERAL SCIENCE Short Answer Fertilizers typically have 3 numbers such as 5, 10, 5. What are the chemical elements that represent these numbers, in any order?

NITROGEN; PHOSPHOROUS; POTASSIUM (ACCEPT: N; P; K)

18) MATH Short Answer Solve the following equation for x: !(x - 3)! = -7 (read as: the absolute value of, parenthesis, x - 3, close parenthesis, equals negative 7)

NO SOLUTION (ACCEPT: EMPTY SET)

23) LIFE SCIENCE Short Answer In the stem of a higher plant, what is the MOST common scientific term for the points where leaves are attached?

NODES

8) GENERAL SCIENCE Short Answer In a 2nd degree burn, what is the name of the fluid that quickly forms beneath the blister?

SERUM (ACCEPT: SEROUS FLUID)

13) EARTH SCIENCE Short Answer Which 2 of the following 5 elements are most abundant in the Earth's crust: iron; nickel; carbon; silicon; oxygen

SILICON AND OXYGEN (in any order)

23) EARTH SCIENCE Short Answer What most commonly happens to the speed of ocean and lake waves as they approach the shallow waters of a shore?

SLOW DOWN

22) PHYSICAL SCIENCE Short Answer Which one of the 4 states of matter is described as having a definite shape and volume?

SOLID

22) LIFE SCIENCE Short Answer The ability to interbreed and produce sexually competent offspring is a classic characteristic of what taxonomical level of organisms?

SPECIES

21) GENERAL SCIENCE Short Answer What is the scientific technique astronomers have most commonly used since the 1800's to analyze stars and other heavenly bodies for the presence of certain elements?

SPECTROSCOPY

2) LIFE SCIENCE Short Answer Name all of the following 4 organisms that are considered to have no true tissues: sponges; flatworms; round worms; rotifers

SPONGES

23) EARTH SCIENCE Short Answer What is the specific name for the very high and very low tides that occur when the Sun and Moon are in line with the Earth?

SPRING TIDES

15) LIFE SCIENCE Short Answer What is the scientific term for the male reproductive structure in plants?

STAMEN (ACCEPT: ANTHER)

18) PHYSICAL SCIENCE Short Answer Name all of the following 4 choices that are natural polymers: starch; ethanol; DNA; dextrose

STARCH AND DNA

22) EARTH SCIENCE Short Answer What is the name of the process that occurs along a convergent plate boundary whereby one of the plates bends and sinks into the mantle?

SUBDUCTION

3) EARTH SCIENCE Short Answer When sulfur dioxide combines with atmospheric water, what acid is ultimately produced that has greatly contributed to the formation of acid rain?

SULFURIC ACID (ACCEPT: AQUEOUS SULFURIC ACID OR H2SO4)

15) LIFE SCIENCE Short Answer What is the MOST common term for the root system in most dicots?

TAP ROOT

25) PHYSICAL SCIENCE Short Answer Name all of the following 3 choices that affect the speed of sound in a substance: temperature; density; elasticity

TEMPERATURE; DENSITY; ELASTICITY (ACCEPT: ALL OF THEM)

8) PHYSICAL SCIENCE Short Answer What is the most common term for the maximum downward speed an object can attain, considering air resistance, when it is in free fall from a few miles up in the atmosphere?

TERMINAL VELOCITY

12) GENERAL SCIENCE Short Answer The coronagraph is used to study what celestial object?

THE SUN

22) LIFE SCIENCE Short Answer What is the specific scientific term for organisms that can grow in high temperature environments, such as in undersea thermal vents?

THERMOPHILES (ACCEPT: THERMOPHILIC ORGANISMS/BACTERIA)

8) LIFE SCIENCE Short Answer What is the common name, not the letter abbreviation, of the nucleotide that is present in DNA but not RNA?

THYMINE

mah); vessel elements

TRACHEIDS; VESSEL ELEMENTS

2) PHYSICAL SCIENCE Short Answer Specific gravity compares the density of various substances to that of what reference substance?

WATER

4) GENERAL SCIENCE Multiple Choice Which of the following pieces of laboratory glassware would be used to MOST accurately measure 1000 milliliters of water: W) 1000 milliliter volumetric flask X) 1000 milliliter beaker Y) 1000 milliliter Erlenmeyer flask Z) two, 500 milliliter Erlenmeyer flasks

W) 1000 MILLILITER VOLUMETRIC FLASK

2) PHYSICAL SCIENCE Short Answer What is the only common substance on Earth that exists naturally in the solid, liquid and gaseous states?

WATER

12) LIFE SCIENCE Multiple Choice Which of the following is the BEST description of the primary function of a protein enzyme: W) a catalyst made of amino acids X) a string of amino acids added to a sugar to make the sugar stronger Y) to store genetic information Z) as a short term form of energy storage

W) A CATALYST MADE OF AMINO ACIDS

9) EARTH SCIENCE Multiple Choice Looking at a weather map of the Northern Hemisphere, which of the following BEST describes the circulation of air around a low pressure system, designated with an "L": W) a counterclockwise flow of air into the center X) a counterclockwise flow of air out of the center Y) a clockwise flow of air into the center Z) a clockwise flow of air out of the center

W) A COUNTERCLOCKWISE FLOW OF AIR INTO THE CENTER

23) EARTH SCIENCE Multiple Choice A caldera is: W) a huge depression formed after a magma chamber collapses X) a long tube where magma pours out onto the surface Y) a lateral vent that spews only gases Z) another name for a fumarole

W) A HUGE DEPRESSION FORMED AFTER A MAGMA CHAMBER COLLAPSES

10) PHYSICAL SCIENCE Multiple Choice Which of the following is NOT true about boiling liquids: W) all boiling liquids are too hot to touch X) the boiling point of pure water at standard pressure is 373.15 kelvin Y) the boiling point of a liquid depends on the surface pressure above the liquid Z) adding a handful of table salt to a quart of boiling water will change its boiling point

W) ALL BOILING LIQUIDS ARE TOO HOT TO TOUCH

17) EARTH SCIENCE Multiple Choice Volcanoes and earthquakes occur primarily in which of the following locations: W) along plate boundaries X) along continental shelves Y) at the North and South Poles Z) around the Hawaiian Islands

W) ALONG PLATE BOUNDARIES

4) GENERAL SCIENCE Multiple Choice Which of the following types of scientists would most likely use interferometry in their experiments: W) astronomer X) plastics engineer Y) chemical engineer Z) microbiologist

W) ASTRONOMER

16) LIFE SCIENCE Multiple Choice What type of virus has the Food and Drug Administration recently approved to be sprayed on certain foods as a preservative against bacterial contamination: W) bacteriophage X) H5N1 Y) Marburg Z) Ebola

W) BACTERIOPHAGE

14) GENERAL SCIENCE Multiple Choice Which of the following stars would have the hottest surface temperature: W) Blue Giant X) Red Supergiant Y) red dwarf Z) brown dwarf

W) BLUE GIANT

11) EARTH SCIENCE Multiple Choice Coral reefs capture about half of all calcium flowing into the ocean every year by fixing it into: W) calcium carbonate X) calcium fluoride Y) calcium oxalate Z) quartz

W) CALCIUM CARBONATE

20) LIFE SCIENCE Multiple Choice Differentiation of cells during plant development is MOST commonly associated with: W) cellular maturation into specialized cells X) enlargement of cells without specialization Y) division of cells without specialization Z) allowing cells to recognize other cells

W) CELLULAR MATURATION INTO SPECIALIZED CELLS

15) GENERAL SCIENCE Multiple Choice Which of the following is NOT true about nicotine: W) chewing or smokeless tobaccos are safe when used properly because nicotine cannot be absorbed through the mucous membrane X) nicotine and cyanide have been found in cigarettes Y) during pregnancy, nicotine concentrates in breast milk and fetal blood Z) nicotine is a plant alkaloid that is poisonous to most animals

W) CHEWING OR SMOKELESS TOBACCOS ARE SAFE WHEN USED PROPERLY BECAUSE NICOTINE CANNOT BE ABSORBED THROUGH THE MUCOUS MEMBRANE

19) GENERAL SCIENCE Multiple Choice Which of the following are sometimes called "big dirty snowballs" because of their composition: W) comets X) asteroids Y) meteorites Z) dwarf planets

W) COMETS

4) EARTH SCIENCE Multiple Choice Which of the following is a period on the Geological Time Scale that is part of the Paleozoic Era: W) Devonian X) Cretaceous Y) Tertiary Z) Quaternary

W) DEVONIAN

9) GENERAL SCIENCE Multiple Choice Which of the following is the MAIN advantage of the LED, or light emitting diode, versus the conventional incandescent lamp: W) efficiency X) size Y) they can be made out of plastic Z) they are cheaper to manufacture

W) EFFICIENCY

18) GENERAL SCIENCE Multiple Choice The majority of sulfuric acid produced in the U.S. today is used to make which of the following: W) fertilizers X) glass Y) paint Z) plastics

W) FERTILIZERS

22) LIFE SCIENCE Multiple Choice What structures on the outside of bacteria are MOST typically associated with the ability of bacteria to control their movement: W) flagella X) cilia Y) pseudopodia Z) spirillus

W) FLAGELLA

24) GENERAL SCIENCE Multiple Choice Which of the following U.S. states is LEAST prone to having earthquakes because it does not lie near an existing fault zone: W) Florida X) Alaska Y) Washington Z) Missouri

W) FLORIDA

3) GENERAL SCIENCE Multiple Choice MTBE or methyl tertiary butyl ether, is a: W) gasoline additive X) organic dye Y) food preservative Z) plant alkaloid

W) GASOLINE ADDITIVE

4) PHYSICAL SCIENCE Multiple Choice In a modern optimally functioning internal combustion engine, most of the energy is lost as: W) heat X) unburnt fuel Y) CO2 Z) carbon monoxide

W) HEAT

2) EARTH SCIENCE Multiple Choice Which of the following is NOT typically true about fossils: W) index fossils are not geographically widespread X) fossilized marine animals have been found in the Himalayan Mountains Y) fossils found in tar pits are very well preserved Z) petrified wood is a type of fossil

W) INDEX FOSSILS ARE NOT GEOGRAPHICALLY WIDESPREAD

17) LIFE SCIENCE Multiple Choice Which of the following is the BEST explanation of why DDT was such a problem for raptors: W) it is stable and accumulates in animals as it moves up the food chain X) it causes extreme light sensitivity and since raptors have no eye lids it causes them to go blind quickly Y) it is highly concentrated in the kidneys of mice which are a main food source of most raptors Z) raptors are extremely sensitive to very low doses of DDT and it causes immediate death on exposure

W) IT IS STABLE AND ACCUMULATES IN ANIMALS AS IT MOVES UP THE FOOD CHAIN

12) PHYSICAL SCIENCE Short Answer What is the common term for the force of gravity that is exerted on an object?

WEIGHT

million year old fossil discovered in the upper reaches of Canada had many unusual features including a flat head with eyes on top and overlapping ribs. This was a major discovery because it is believed this: W) represents a missing link between fish and land vertebrates X) was the first juvenile Tyrannosaurus ever found in Canada Y) was an amphibian bird from the Cretaceous Period Z) supports vertebrates evolving first on land rather than at sea

W) REPRESENTS A MISSING LINK BETWEEN FISH AND LAND VERTEBRATES

7) EARTH SCIENCE Multiple Choice Which of the following primarily influences the viscosity and therefore the explosivity of a magma: W) silica content X) gas content Y) temperature Z) pressure

W) SILICA CONTENT

3) PHYSICAL SCIENCE Multiple Choice Which of the following BEST describes the following reaction: 4Fe + 3O2 ! 2Fe2O3 W) synthesis X) combustion Y) single replacement Z) double replacement

W) SYNTHESIS

15) GENERAL SCIENCE Multiple Choice In which of the following biomes will there be trees whose leaves change color in autumn, fall off in winter, and grow back in the spring: W) temperate deciduous forest X) rainforest Y) coniferous forest Z) scrubland

W) TEMPERATE DECIDUOUS FOREST

4) EARTH SCIENCE Multiple Choice The Moon is full on September 7, 2006. On which of the following days that month will a last quarter Moon occur: W) the 14th X) the 21st Y) the 28th Z) the 29th

W) THE 14TH

9) EARTH SCIENCE Multiple Choice Which of the following is NOT true of tornados in the U.S.: W) the funnel cloud typically develops out of a nimbostratus cloud X) their strength is measured with Fujita Scale Y) supercells are the most dangerous and violent tornadoes in the U.S. Z) wind speeds can reach an excess of 200 mph

W) THE FUNNEL CLOUD TYPICALLY DEVELOPS OUT OF A NIMBOSTRATUS CLOUD

19) EARTH SCIENCE Multiple Choice In which of the following ways is firn formed on glaciers: W) the partial melting and refreezing of snow crystals into ice grains X) the polishing of rocks and boulders by abrasion in stream beds Y) layers of sand and sediment originating in glaciers that are transported upslope by strong winds Z) friction between bedrock along the base of a glacier

W) THE PARTIAL MELTING AND REFREEZING OF SNOW CRYSTALS INTO ICE GRAINS

4) GENERAL SCIENCE Multiple Choice If you draw a triangle on a sphere such as a basketball, which of the following is TRUE: W) the sum of the 3 angles is greater than 180º X) the sum of the 3 angles is equal to 180º Y) the sum of the 3 angles is less than 180º Z) the triangle has negative curvature and is not considered a triangle

W) THE SUM OF THE 3 ANGLES IS GREATER THAN 180º

6) EARTH SCIENCE Multiple Choice Which of the following is TRUE about Earth's atmosphere: W) the upper mesosphere is the coldest region of the Earth's atmosphere X) the stratopause lies just below the stratosphere Y) moist air weighs more than dry air at the same temperature Z) the tropopause is higher at the South Pole than at the equator

W) THE UPPER MESOSPHERE IS THE COLDEST REGION OF THE EARTH'S ATMOSPHERE

17) PHYSICAL SCIENCE Multiple Choice Which of the following follows from the Doppler effect for a sound wave source where a loud motorcycle is moving towards you: W) the wavelength will shorten and frequency increases X) the wavelength will shorten and frequency decreases Y) the wavelength will increase and frequency increases Z) the wavelength will increase and frequency decreases

W) THE WAVELENGTH WILL SHORTEN AND FREQUENCY INCREASES

13) EARTH SCIENCE Multiple Choice Which of the following is the name for the last of 4 glacial episodes recognized in North America: W) the Wisconsinan X) the Ordovician Y) the Alaskan Z) the Nebraskan

W) THE WISCONSINAN

19) GENERAL SCIENCE Multiple Choice Which of the following is NOT true about comets: W) to Earth-bound observers, comets glow because they are nearing Earth's atmosphere X) they typically have elliptical orbits Y) the Deep Impact probe recently struck a comet Z) the solid part of a comet is called the nucleus

W) TO EARTH-BOUND OBSERVERS, COMETS GLOW BECAUSE THEY ARE NEARING EARTH'S ATMOSPHERE

17) EARTH SCIENCE Multiple Choice Which of the following is the primary process by which colder ocean water loaded with nutrients rises to the surface: W) upwelling X) ocean swells Y) maelstroms Z) eddies

W) UPWELLING

14) EARTH SCIENCE Multiple Choice Which of the following is NOT true about magma: W) volcanic glass results when magma cools very slowly X) fluid basaltic magma typically produces the least pyroclastic material Y) granitic magma contains more silica and water than basaltic magma Z) andesitic magma is dominant in volcanoes around the Pacific Ocean Basin

W) VOLCANIC GLASS RESULTS WHEN MAGMA COOLS VERY SLOWLY

10) EARTH SCIENCE Multiple Choice A hurricane originating in the Eastern tropical Atlantic is driven: W) westward by the easterly trade winds X) westward by the prevailing westerlies Y) eastward by the easterly trade winds Z) eastward by the prevailing westerlies

W) WESTWARD BY THE EASTERLY TRADE WINDS

8) PHYSICAL SCIENCE Multiple Choice A litmus paper test is used to determine: W) whether a solution is acidic or basic X) simple metallic elements Y) rates of reactions Z) bacterial counts in drinking water

W) WHETHER A SOLUTION IS ACIDIC OR BASIC

7) GENERAL SCIENCE Multiple Choice Which of the following fabrics would most easily be colored by dyes that contain polar molecules or ions: W) wool X) nylon Y) polyester Z) rayon

W) WOOL (Solution: wool being a protein has the most polar molecules)

23) EARTH SCIENCE Multiple Choice Which of the following sea creatures is LEAST likely to be living in the intertidal shore along the state of Maine's rocky coast: W) zebra mussel X) barnacle Y) anemone Z) limpet

W) ZEBRA MUSSEL

15) MATH Multiple Choice Which of the following equations has a graph that is a circle: W) x 2 + y 2 = 25 X) 4y = 3x + 2 Y) y = x 2 Z) x - 3 = 0

W) x 2 + y 2 = 25

18) MATH Multiple Choice Which of the following is the equation of a parabola whose graph opens downward: W) y = -x 2 X) y = x 3 Y) y = x 2 + 2 Z) y = x 2 - 2

W) y = -x 2

2) EARTH SCIENCE Short Answer What molecular substance is primarily responsible for moderating wide changes in temperatures on Earth?

WATER

16) LIFE SCIENCE Multiple Choice Which of the following is the BEST reason why high salt environments are difficult environments for organisms to survive in: W) salt is an ionic molecule and causes a charge differential X) salt causes an osmotic imbalance leading to excessive loss of water from cells Y) salt usually is contaminated with toxic heavy metals Z) salt causes heavy metal poisons to be readily absorbed through skin

X) SALT CAUSES AN OSMOTIC IMBALANCE LEADING TO EXCESSIVE LOSS OF WATER FROM CELLS

identical twins X) the matching alleles for a certain physical characteristic are different Y) one of a pair of chromosomes is lost Z) two zygotes are fused together

X) THE MATCHING ALLELES FOR A CERTAIN PHYSICAL CHARACTERISTIC ARE DIFFERENT

22) PHYSICAL SCIENCE Multiple Choice According to thermodynamics, whenever energy is converted from one form to another then: W) the net useful energy remains the same X) the net useful energy must decrease Y) the total energy of the system increases Z) the total energy of the system decreases

X) THE NET USEFUL ENERGY MUST DECREASE

16) PHYSICAL SCIENCE Multiple Choice Which of the following occurs as a bottle of milk falls from the edge of a table until just before it strikes a floor: W) all the bottle's potential energy is lost to heat and friction X) the potential energy of the bottle decreases Y) the potential energy of the bottle increases Z) the total potential energy of the bottle remains the same

X) THE POTENTIAL ENERGY OF THE BOTTLE DECREASES

21) PHYSICAL SCIENCE Multiple Choice The octet rule MOST directly involves which of the following: W) only atoms with 8 or more protons X) the sharing of valence electrons Y) the balance of neutrons to protons Z) the loss of protons in radioactive decay

X) THE SHARING OF VALENCE ELECTRONS

20) GENERAL SCIENCE Multiple Choice Earth's seasons are primarily caused by: W) Earth's distance from the Sun X) the tilt of Earth's axis with respect to the Sun Y) Earth's elliptical orbit around the Sun Z) Earth's rotational period

X) THE TILT OF EARTH'S AXIS WITH RESPECT TO THE SUN

causing substances, which makes them excellent to observe evolving under stress

X) THEY ARE CHEAP TO MAINTAIN, PRODUCE MANY OFFSPRING, AND HAVE A SHORT TIME BETWEEN GENERATIONS

21) LIFE SCIENCE Multiple Choice Which of the following is the most basic function of human eye lashes: W) to shade the eye from bright sun X) to prevent small particles from entering the eyes Y) to attract a mate Z) to help disperse tears when blinking

X) TO PREVENT SMALL PARTICLES FROM ENTERING THE EYES

11) LIFE SCIENCE Multiple Choice If an order of plant has floral parts usually arranged in multiples of 4 or 5, it would most likely have: W) net venation and one cotyledon X) vascular bundles arranged in a ring and a taproot Y) diffuse or fibrous root system and 2 cotyledons Z) parallel venation and 2 cotyledons

X) VASCULAR BUNDLES ARRANGED IN A RING AND A TAPROOT

6) GENERAL SCIENCE Multiple Choice At which of the following locations would you find the intersection of 0º longitude and 0º latitude: W) where the International Date Line crosses the equator X) where the prime meridian crosses the equator Y) along the 180th meridian Z) at the North and South Poles

X) WHERE THE PRIME MERIDIAN CROSSES THE EQUATOR

18) LIFE SCIENCE Multiple Choice In which of the following cell types of woody higher plants is lignin most prominent: W) phloem X) xylem Y) parenchyma Z) collenchyma

X) XYLEM

21) GENERAL SCIENCE Multiple Choice Buffers in biological fluids primarily work to resist changes in: W) temperature X) pH Y) toxins Z) osmotic pressure

X) pH

9) EARTH SCIENCE Multiple Choice Which of the following is closest to average barometric pressure at sea level, in inches: W) 1013 X) 10.13 Y) 29.92 Z) 27.72

Y) 29.92

20) GENERAL SCIENCE Multiple Choice Which of the following BEST approximates the circumference of a U.S. nickel: W) 1.41 centimeters X) 3.14 centimeters Y) 6.28 centimeters Z) 9.42 centimeters

Y) 6.28 CENTIMETERS (Solution: C = π(2cm) = 6.28cm)

10) MATH Multiple Choice Which of the following numbers is evenly divisible by 4: W) 174 X) 414 Y) 636 Z) 1,126

Y) 636 (Solution: 36, the last 2 digits, is a multiple of 4)

13) LIFE SCIENCE Multiple Choice A small human lymphocyte and a red blood cell are both about how many micrometers across: W) 1 X) 3 Y) 8 Z) 16

Y) 8

25) MATH Multiple Choice Which of the following is NOT true about polygons: W) the sum of the exterior angles of any polygon equals 720º X) a regular polygon is always equilateral Y) a square is not a regular polygon Z) a regular pentagon is a polygon with 5 congruent angles and 5 congruent sides

Y) A SQUARE IS NOT A REGULAR POLYGON

16) PHYSICAL SCIENCE Multiple Choice Which of the following is NOT true: W) sound waves can be considered longitudinal waves X) light slows down when it enters glass from dry air Y) as a light wave's frequency increases so must its amplitude Z) light can be produced with one frequency

Y) AS A LIGHT WAVE'S FREQUENCY INCREASES SO MUST ITS AMPLITUDE

20) LIFE SCIENCE Multiple Choice Which of the following is the MOST readily available energy store during normal human cellular conditions: W) fat X) proteins Y) ATP Z) glucose

Y) ATP

10) GENERAL SCIENCE Multiple Choice Honey is considered hygroscopic because it: W) is very dense for a sugar X) loses water to its environment Y) attracts water from the air Z) expands rapidly on heating

Y) ATTRACTS WATER FROM THE AIR

12) EARTH SCIENCE Multiple Choice Which of the following is NOT true about glaciations: W) about 15,000 years ago, most of Wisconsin was buried under thick glacial ice X) throughout history, times such as now when only minor ice sheets existed coincided with high sea levels Y) because of its high altitude, Mt. Everest contains a vast continental glacier Z) Australia does not currently contain any glaciers

Y) BECAUSE OF ITS HIGH ALTITUDE, MT. EVEREST CONTAINS A VAST CONTINENTAL GLACIER

16) GENERAL SCIENCE Multiple Choice If you were told you were anemic and had to increase your dietary iron uptake, which of the following foods would provide the most iron: W) green beans X) spinach Y) beef spleen Z) broiled chicken thighs

Y) BEEF SPLEEN

13) LIFE SCIENCE Multiple Choice Which of the following kinds of microscopes typically is used to view the surface of objects ranging in size from cells to small insects like fleas to reveal minute surface details: W) bright field microscopy X) interference microscopy Y) scanning electron microscopy Z) transmission electron microscopy

Y) SCANNING ELECTRON MICROSCOPY

1) PHYSICAL SCIENCE Multiple Choice When magnesium and zinc bromide are combined, zinc and magnesium bromide are formed according to the reaction, Mg + ZnBr2 ! Zn + MgBr2. Which of the following types of reactions has occurred: W) synthesis X) combustion Y) single replacement Z) double replacement

Y) SINGLE REPLACEMENT

2) LIFE SCIENCE Multiple Choice Which of the following is a reptile: W) blue poison arrow frog X) sea lamprey Y) snapping turtle Z) salamander

Y) SNAPPING TURTLE

9) PHYSICAL SCIENCE Multiple Choice Which of the following is a Group 1 metal that gives off a persistent yellow color when burned in a flame test, and is also used in commercial and highway vapor lamps that have a yellow glow: W) neon X) potassium Y) sodium Z) zinc

Y) SODIUM

14) PHYSICAL SCIENCE Multiple Choice Which of the following choices is NOT an example of a chemical change: W) shiny new copper tarnishing to a green color X) bubbles forming in a solution of vinegar following the addition of baking soda Y) steel turning red upon heating Z) iron rusting

Y) STEEL TURNING RED UPON HEATING

16) PHYSICAL SCIENCE Multiple Choice Which of the following is the scientific term for a drink that settles and needs to be shaken to redistribute the particles throughout the drink: W) homogenous X) solvent Y) suspension Z) colloid

Y) SUSPENSION

18) EARTH SCIENCE Multiple Choice The size, shape and arrangement of crystals or mineral grains in igneous rocks is called: W) hardness X) density Y) texture Z) streak

Y) TEXTURE

19) GENERAL SCIENCE Multiple Choice Which of the following BEST explains why when a balloon filled with air is moved from a cold room to a warm room, the balloon expands: W) the air pressure on the outside of the balloon is now greater X) the air molecules inside the balloon get bigger Y) the air molecules inside the balloon move faster Z) the room's air molecules are repelled from the outside of the balloon by electrostatic forces

Y) THE AIR MOLECULES INSIDE THE BALLOON MOVE FASTER

20) EARTH SCIENCE Multiple Choice Which of the following is NOT true about glaciations: W) about 12,000 years ago it was possible to walk from Alaska to Siberia through the Bering Strait X) throughout history, times of extensive glaciation coincided with low sea levels Y) the center of a glacier typically moves slower down a slope than do the edges Z) alpine glaciers exist on nearly every continent

Y) THE CENTER OF A GLACIER TYPICALLY MOVES SLOWER DOWN A SLOPE THAN DO THE EDGES

9) GENERAL SCIENCE Multiple Choice Which of the following must occur for total internal reflection to occur: W) the angle of incidence equals the angle of reflection X) dispersion of all wavelengths of light Y) the critical angle of refraction is exceeded Z) polarized light must be used

Y) THE CRITICAL ANGLE OF REFRACTION IS EXCEEDED

20) PHYSICAL SCIENCE Multiple Choice During normal chemical reactions, although the combinations of atoms that form molecules can change, the total number of atoms must remain the same. This is called: W) the law of interchange X) the atomic principle Y) the law of conservation of matter Z) Dalton's law

Y) THE LAW OF CONSERVATION OF MATTER

2) GENERAL SCIENCE Multiple Choice Which of the following is NOT true of the Moon: W) the Moon rotates on its axis at the same rate that it takes to orbit the Earth X) over long periods of time, we can see more than 50% of the Moon's surface from Earth Y) the Moon is gradually getting closer to the Earth by about 4 centimeters each year Z) the Moon's orbit is elliptical

Y) THE MOON IS GRADUALLY GETTING CLOSER TO THE EARTH BY ABOUT 4 CENTIMETERS EACH YEAR

5) GENERAL SCIENCE Multiple Choice Which of the following is a MAIN advantage of a Newtonian reflector telescope over a Galilean refractor telescope: W) the Newtonian is much smaller and resistant to changes in temperature X) the Newtonian can be used in a wider variety of temperatures Y) the Newtonian focuses all colors of light at the same focal point Z) the Newtonian mirrors are usually made of metal so they cannot easily break

Y) THE NEWTONIAN FOCUSES ALL COLORS OF LIGHT AT THE SAME FOCAL POINT

8) PHYSICAL SCIENCE Multiple Choice Which of the following BEST explains where the bubbles in a boiling pot of water come from: W) the water is condensing X) hydrogen gas is produced and it rises quickly through the water Y) the water is vaporizing Z) oxygen from the atmosphere enters the water

Y) THE WATER IS VAPORIZING

11) PHYSICAL SCIENCE Multiple Choice Which of the following is the BEST description of the arrangement of the magnetic domains of an unmagnetized iron bar: W) all are aligned at 180º to each other X) all are aligned at 90º to each other Y) they are randomly arranged Z) there are no magnetic domains in this case

Y) THEY ARE RANDOMLY ARRANGED

16) EARTH SCIENCE Multiple Choice Which of the following BEST explains why there is typically less temperature variation along the coasts of continents versus inland climates at the same latitude: W) land surfaces of the planet can store more heat than the oceans X) the friction between waves and the shore add heats to the area Y) water can absorb and release large amounts of heat with very little change in temperature Z) surface ocean currents along shorelines typically carry warm water

Y) WATER CAN ABSORB AND RELEASE LARGE AMOUNTS OF HEAT WITH VERY LITTLE CHANGE IN TEMPERATURE

3) PHYSICAL SCIENCE Multiple Choice Which of the following would MOST likely occur to a sample of water if it underwent an increase in atmospheric pressure from 1 atmosphere to 5 atmospheres: W) its mass would increase and its temperature would rise by about 5ºC X) its mass would increase and its volume would decrease by about 5% Y) its mass would decrease and its volume would decrease by about 25% Z) its mass would remain the same and its volume would remain the same

Z) ITS MASS WOULD REMAIN THE SAME AND ITS VOLUME WOULD REMAIN THE SAME

18) PHYSICAL SCIENCE Multiple Choice Which of the following is NOT true: W) weight decreases as you move away from the Earth's surface X) mass times gravitational acceleration is weight Y) gravitational potential energy increases with increasing height Z) kinetic energy can only decrease when potential energy decreases

Z) KINETIC ENERGY CAN ONLY DECREASE WHEN POTENTIAL ENERGY DECREASES

11) LIFE SCIENCE Multiple Choice Which of the following is closest to the length of time it takes for a typical human cell to complete mitosis: W) 1 second X) 1 minute Y) 1 hour Z) 1 day

Z) 1 DAY

17) MATH Multiple Choice Which of the following is equivalent to the radical expression, (18 ) 18 Read as: square root of open parenthesis 18 to the 18th power closed parenthesis W) 33 X) 99 Y) 183 Z) 189

Z) 189

20) EARTH SCIENCE Multiple Choice If high tide occurs at noon, which of the following is the most likely time that the same beach will experience a low tide: W) 3 PM X) 4 PM Y) 5 PM Z) 6 PM

Z) 6 PM

16) PHYSICAL SCIENCE Multiple Choice Which of the following is NOT true about phase changes in substances: W) energy is exchanged during a phase change X) sublimation is a type of phase change involving solids Y) freezing is an exothermic phase change Z) a phase change cannot be reversed

Z) A PHASE CHANGE CANNOT BE REVERSED

ray telescope Y) an adaptive optical infrared telescope Z) a radio telescope

Z) A RADIO TELESCOPE

1) GENERAL SCIENCE Multiple Choice Which of the following is NOT true about solutions: W) ethylene glycol, or antifreeze, lowers the freezing point of water in a car radiator X) reverse osmosis is used to convert salt water into fresh water Y) the presence of colloidal particles in solution is easily detected by a beam of light Z) a solution of hydrochloric acid acts as an ideal proton acceptor

Z) A SOLUTION OF HYDROCHLORIC ACID ACTS AS AN IDEAL PROTON ACCEPTOR

14) GENERAL SCIENCE Multiple Choice Which of the following statements is NOT true: W) yeast reproduces asexually X) lichens are the mainstay of the caribou diet Y) coal is not a native mineral Z) a spider's web is made from a specialized carbohydrate

Z) A SPIDER'S WEB IS MADE FROM A SPECIALIZED CARBOHYDRATE

3) LIFE SCIENCE Multiple Choice Antibodies are composed of: W) lipids X) nucleic acids Y) glucose Z) amino acids

Z) AMINO ACIDS

18) MATH Multiple Choice Which of the following is NOT true: W) two numbers are reciprocals if their product is 1 X) one-half of three quarters is three eighths Y) a whole number is divisible by 5 if the 'ones' digit is 0 or 5 Z) an example of a proper fraction is 6/4

Z) AN EXAMPLE OF A PROPER FRACTION IS 4 6

18) MATH Multiple Choice Which of the following is NOT true: W) two numbers are reciprocals if their product is 1 X) one-half of three quarters is three eighths Y) a whole number is divisible by 5 if the 'ones' digit is 0 or 5 Z) an example of a proper fraction is 6/4

Z) AN EXAMPLE OF A PROPER FRACTION IS 6/4

20) LIFE SCIENCE Multiple Choice Which of the following is the location of the biceps humoris muscle: W) chest X) leg Y) abdomen Z) arm

Z) ARM

16) LIFE SCIENCE Multiple Choice To which of the following categories of living organisms does E. coli belong: W) viruses X) protozoans Y) fungi Z) bacteria

Z) BACTERIA

7) GENERAL SCIENCE Multiple Choice There are about 24 essential elements needed to maintain a healthy human body. Which of the following is NOT one of these elements: W) iodine X) selenium Y) sodium Z) cesium

Z) CESIUM

15) GENERAL SCIENCE Multiple Choice You wake up in the morning and your eyelids are stuck together with a thin or thick discharge caused by inflammation of the clear membrane that covers the white part of your eyes. Which of the following is MOST likely your problem: W) sinusitis X) cellulitis Y) osteomyelitis Z) conjunctivitis

Z) CONJUNCTIVITIS

4) GENERAL SCIENCE Multiple Choice Which of the following practices is one of the MAIN ways of preventing Brucellosis in humans: W) avoid tick bites X) include vitamin C in your diet Y) include niacin in your diet Z) consume only pasteurized dairy products

Z) CONSUME ONLY PASTEURIZED DAIRY PRODUCTS

15) EARTH SCIENCE Multiple Choice Which of the following is immediately released when an earthquake occurs: W) convection currents within the mantle X) radiation energy originating from the mantle Y) stored heat energy from the Sun Z) elastic energy stored in rocks

Z) ELASTIC ENERGY STORED IN ROCKS

13) GENERAL SCIENCE Multiple Choice It is believed that the heavy elements in Earth's crust primarily originated from: W) comets X) asteroids Y) meteorites Z) exploding stars called supernovae

Z) EXPLODING STARS CALLED SUPERNOVAE

14) EARTH SCIENCE Multiple Choice Which of the following is NOT true about rocks: W) corundum is harder than quartz X) pyrite is also known as 'fool's gold' Y) halite occurs as a perfect cubic crystal Z) felsic rocks contain large amounts of magnesium and iron

Z) FELSIC ROCKS CONTAIN LARGE AMOUNTS OF MAGNESIUM AND IRON

25) EARTH SCIENCE Multiple Choice Which of the following is NOT directly or indirectly a result of a large earthquake: W) fires X) tsunami Y) mass wasting Z) lightning

Z) LIGHTNING

eye, which of the following diseases would you expect you have: W) malaria X) poison ivy Y) sickle cell anemia Z) Lyme disease

Z) LYME DISEASE BONUS 14) GENERAL SCIENCE Multiple Choice Which of the following is NOT true about Lyme disease: W) only a small percentage of tick bites in the U.S. result in Lyme disease X) in early stages, people experience flu

15) LIFE SCIENCE Multiple Choice Which of the following is the name for the blood cell in humans that plays the most central role in regulating the immune system: W) macrophage X) mast cell Y) erythrocyte Z) lymphocyte

Z) LYMPHOCYTE

25) EARTH SCIENCE Multiple Choice The area known as the 'ring of fire' is prone to intense earthquake and volcanic activity because this is where: W) the oceanic crust is the thinnest X) the oceanic crust is the densest Y) density differences between the lithosphere and the mantle are most pronounced Z) most convergent plate boundaries are found

Z) MOST CONVERGENT PLATE BOUNDARIES ARE FOUND

22) PHYSICAL SCIENCE Multiple Choice Which of the following is the BEST example of a suspension: W) water with 10% ethanol added to it X) olive oil in water Y) table sugar completely dissolved in water Z) orange juice with pulp

Z) ORANGE JUICE WITH PULP

19) EARTH SCIENCE Multiple Choice Meandering streams are most commonly noted to form which of the following specific bodies of water: W) high mountain stagnant pond X) fen Y) bog Z) oxbow lake

Z) OXBOW LAKE BONUS 19) EARTH SCIENCE Multiple Choice Which of the following is believed to create Earth's magnetic field: W) the tilt in Earth's axis rocking Earth's inner core X) the spinning of Earth's outer core Y) the Coriolis effect Z) how fast the Earth revolves around the Sun ANSWER: X) THE SPINNING OF EARTH'S OUTER CORE TOSS

17) GENERAL SCIENCE Multiple Choice Which of the following gases was most likely LEAST abundant in Earth's early atmosphere: W) carbon dioxide X) ammonia Y) water Z) oxygen

Z) OXYGEN

7) EARTH SCIENCE Multiple Choice Which of the following is a gas that is a pollutant when present in the troposphere but beneficial when present in the stratosphere: W) nitrogen X) chlorofluorocarbons Y) hydrogen Z) ozone

Z) OZONE

15) PHYSICAL SCIENCE Multiple Choice Which of the following is MOST commonly used to separate substances based on solubility differences: W) centrifugation X) autoclaving Y) paper filtration Z) paper chromatography

Z) PAPER CHROMATOGRAPHY

9) EARTH SCIENCE Multiple Choice Which of the following is the global wind system that most influences weather between 60º and 90º north and south latitudes: W) tropical easterlies X) prevailing westerlies Y) trade winds Z) polar easterlies

Z) POLAR EASTERLIES

17) GENERAL SCIENCE Multiple Choice Which of the following BEST describes the property of light as a transverse wave whereby the plane of vibration of the light is restricted to certain planes: W) diffraction X) reflection Y) refraction Z) polarization

Z) POLARIZATION

17) EARTH SCIENCE Multiple Choice Which of the following BEST describes or exemplifies an aquifer: W) superheated water bursting through surface cracks in the Earth X) a lake, pond or stream Y) the side of a mountain facing the prevailing winds Z) porous subsurface rock saturated with water

Z) POROUS SUBSURFACE ROCK SATURATED WITH WATER

20) PHYSICAL SCIENCE Multiple Choice Raising a bucket of chicken fat from the floor to 3 meters high off the floor would increase the bucket's: W) kinetic energy X) chemical energy Y) thermal energy Z) potential energy

Z) POTENTIAL ENERGY

7) PHYSICAL SCIENCE Multiple Choice Doing a given amount of work at different rates always involves a different amount of: W) mass X) efficiency Y) potential energy Z) power

Z) POWER

18) EARTH SCIENCE Multiple Choice Eutrophic lakes are most commonly noted for which of the following sets of characteristics: W) high temperature differences from top to bottom and extremely deep X) unhealthy containing no living organisms and a very low pH Y) healthy with abundant life and neutral pH Z) shallow, warm, and containing abundant nutrients

Z) SHALLOW, WARM, AND CONTAINING ABUNDANT NUTRIENTS

15) LIFE SCIENCE Multiple Choice Which of the following are the sugar transporting cells of the phloem in flowering plants: W) tracheids X) collenchyma Y) vessel elements Z) sieve tube cells

Z) SIEVE TUBE CELLS

18) PHYSICAL SCIENCE Multiple Choice Potential energy is most synonymous with: W) kinetic energy X) mechanical energy Y) work energy Z) stored energy

Z) STORED ENERGY

13) PHYSICAL SCIENCE Multiple Choice Which of the following statements is NOT true: W) neutral pH solutions have small but equal numbers of hydronium and hydroxide ions X) when an acid is added to water it will increase the solutions concentration of hydronium ions Y) strong acids dissociate almost completely in water Z) strong bases do not dissociate in water

Z) STRONG BASES DO NOT DISSOCIATE IN WATER

13) PHYSICAL SCIENCE Multiple Choice Which of the following is NOT an element: W) tin X) lead Y) radon Z) sulfate

Z) SULFATE

20) EARTH SCIENCE Multiple Choice Which of the following is NOT true about Earth's water: W) the Antarctic ice sheet holds most of the world's fresh water X) an estuary is a place where a fresh water river is mixed with ocean water Y) rivers and lakes make up less than 1% of all fresh water Z) the global ocean conveyor system is powered mostly by surface winds

Z) THE GLOBAL OCEAN CONVEYOR SYSTEM IS POWERED MOSTLY BY SURFACE WINDS

5) LIFE SCIENCE Multiple Choice Gary the gardener raises a species of annual flowers that can produce pink or white flowers. Gary buys a pot of white flowers and plants them in his garden and each successive year saves only that year's generation of seeds to plant the next year. Over many generations of years of flowers Gary only ever sees white flowers. Which of the following is the BEST explanation for this breeding pattern: W) the original plant was homozygous dominant X) the original plant was homozygous recessive Y) the original plant was heterozygous for both colors Z) the original plant was homozygous but Gary cannot know for sure whether it is a dominant or recessive color

Z) THE ORIGINAL PLANT WAS HOMOZYGOUS BUT GARY CANNOT KNOW FOR SURE WHETHER IT IS A DOMINANT OR RECESSIVE COLOR

12) EARTH SCIENCE Multiple Choice Which of the following is NOT true about Earth's atmosphere: W) carbon monoxide is found in trace amounts in Earth's atmosphere at sea level X) nitrogen and oxygen make up nearly 99% of Earth's atmosphere at sea level Y) temperatures in parts of the stratosphere are nearly as warm as they are on Earth Z) the tropopause is lower at the equator than at the north pole

Z) THE TROPOPAUSE IS LOWER AT THE EQUATOR THAN AT THE NORTH POLE

6) LIFE SCIENCE Multiple Choice Which of the following types of microscopes passes negatively charged particles guided by magnetic lenses through a very thin specimen, all in a partial vacuum, to produce an image at high magnification: W) bright field microscopy X) scanning electron microscopy Y) dark field microscopy Z) transmission electron microscopy

Z) TRANSMISSION ELECTRON MICROSCOPY

10) LIFE SCIENCE Multiple Choice Which of the following is the BEST description for the location of the human spleen: W) lower right abdomen just above the hip bone X) upper right abdomen just under the lower right ribs Y) lower left abdomen just above the hip bone Z) upper left abdomen just under the lower left ribs

Z) UPPER LEFT ABDOMEN JUST UNDER THE LOWER LEFT RIBS

oxidant vitamin typically found in relatively high amounts in vegetable oils, nuts and dark green leafy vegetables: W) vitamin K X) vitamin C Y) vitamin D Z) vitamin E

Z) VITAMIN E


Kaugnay na mga set ng pag-aaral

Biology 224: Chapter 23 (Connect) Metabolism & Nutrition

View Set

Starting Out with Python, 3e Ch 12

View Set

Ch 44 Thyroid and Antithyroid Drugs

View Set

9.15 Windows Boot Process & Startup Error Fixes

View Set